Path Exam 4 Review Questions

Lakukan tugas rumah & ujian kamu dengan baik sekarang menggunakan Quizwiz!

A 33-year-old woman has been bothered by a bump on the dorsum of her left wrist for the past 4 months. On physical examination, there is a 1-cm firm but fluctuant subcutaneous nodule over an extensor tendon of the left wrist. The nodule is painful on palpation and movement. Mucoid fluid is aspirated from the nodule. What is the most likely diagnosis? A Ganglion cyst B Giant cell tumor C Lipoma D Nodular fasciitis E Rheumatoid nodule F Tophus

A A ganglion cyst has a thin wall and clear, mucoid content. It arises in the connective tissue of a joint capsule or tendon sheath. The extensor surfaces of the hands and feet are the most common sites, particularly the wrist. Ganglion cysts probably arise after trauma from focal myxoid degeneration of connective tissue to produce a cystic space. They may regress. If not, and if they are painful, they can be excised. Tenosynovial giant cell tumor (villonodular synovitis) is a more diffuse form of giant cell tumor of tendon sheath (a solid mass lesion) and is a proliferation of mononuclear cells resembling synoviocytes. A lipoma is a mass of adipocytes and is not cystic. Nodular fasciitis is a solid reactive fibroblastic proliferation seen in the upper extremities and trunk of young adults, sometimes occurring after trauma. Rheumatoid nodules are firm, solid masses that typically occur in individuals who already have joint involvement with rheumatoid arthritis. A tophus is a solid mass of chalky sodium urate crystals in patients who have a history of gout.

A 52-year-old woman notes several small, baglike lesions that have appeared on the skin in front of her armpits over the past 2 years. They are located where her bra rubs against the skin. On physical examination, five small, soft papules in the anterior axillary line are covered by wrinkled skin and attached to the skin surface by a thin pedicle. They are 0.5 to 0.8 cm in length and about 0.3 cm in diameter. One lesion has undergone torsion and is more erythematous and painful to touch than the others. What is the most likely diagnosis? A Fibroepithelial polyp B Hemangioma C Lentigo senilis D Melasma E Pilar cyst F Xanthoma

A Also known as an acrochordon or skin tag, a fibroepithelial polyp is a common lesion composed of a central core of fibrovascular connective tissue covered by normal-appearing squamous epithelium. They may become irritated, but are otherwise incidental findings. A hemangioma forms a red nodule that is composed of vascular spaces in the upper dermis. A lentigo is a focus of melanocyte hyperplasia that produces a brown macule; in older individuals, lentigines are known as senile lentigines or age spots and are commonly found on the hands. Melasma is most often a masklike area of facial hyperpigmentation associated with pregnancy. A pilar (trichilemmal) cyst is an epithelial cyst that forms on the scalp. A xanthoma is a localized dermal collection of lipid-laden macrophages associated with hyperlipidemia.

A 31-year-old woman tries a new facial cosmetic product and a day later has red papules from 0.2 to 0.5 cm scattered over her skin. She sees her traditional healer 2 days later, and on physical examination some of the lesions are now vesicular and oozing clear fluid, whereas others are crusted. An herbal preparation is applied over the next 3 days and the lesions subside. Which of the following is the most likely diagnosis for this woman? A Eczematous dermatitis B Erythema multiforme C Impetigo D Lichen simplex chronicus E Urticaria

A An acute eczematous dermatitis is a common reaction to many skin irritants. The epidermis has prominent spongiosis. If the irritant is removed, the lesions resolve in a matter of days. Contact dermatitis is one form that occurs following prior exposure and sensitization. Did the herbal remedy help? Because the lesions will subside on their own, it is hard to prove. If remedies are not harmful and not expensive, then they can have benefit from a placebo effect, which can be powerful (and without adverse reactions). Traditional healers tend to spend more time with patients than do physicians, and if they can sort out the serious problems requiring pharmacologic or surgical approaches, then there is benefit for patient well-being. Erythema multiforme may also be a reaction to a chemical, often a drug, but it is a systemic effect, and the lesions often begin with a targetoid appearance. Impetigo often appears on the face, and the lesions crust, but it is due to an infection (usually staphylococci or streptococci). Eczema may turn into lichen simplex chronicus if the irritant is not removed and there is considerable scratching and rubbing of itchy lesions. Urticaria (hives) appears as wheals from a type I hypersensitivity response, typically from an insect bite or ingested food or substance.

A 50-year-old man has had a chronic cough for the past 18 months. Physical examination shows nasopharyngeal ulcers, and the lungs have diffuse crackles bilaterally on auscultation. Laboratory studies include a serum urea nitrogen level of 75 mg/dL and a creatinine concentration of 6.7 mg/dL. Urinalysis shows 50 RBCs per high-power field and RBC casts. His serologic titer for C-ANCA (proteinase 3) is elevated. A chest radiograph shows multiple, small, bilateral pulmonary nodules. A transbronchial lung biopsy specimen shows a necrotizing inflammatory process involving the small peripheral pulmonary arteries and arterioles. Which of the following is the most likely diagnosis? A Granulomatosis with polyangiitis B Fibromuscular dysplasia C Granuloma pyogenicum D Kaposi sarcoma E Polyarteritis nodosa F Takayasu arteritis

A Anti-neutrophil cytoplasmic autoantibody (ANCA)-associated vasculitis (granulomatosis with polyangiitis) is a form of hypersensitivity reaction to an unknown antigen characterized by necrotizing granulomatous inflammation that typically involves small to medium-sized vessels, although many organ sites may be affected. Pulmonary and renal involvement can be life-threatening. C-ANCAs (antibodies mainly directed against neutrophil proteinase 3) are found in more than 90% of cases. Fibromuscular dysplasia is a hyperplastic medial disorder, usually involving renal and carotid arteries; on angiography, it appears as a "string of beads" caused by thickened fibromuscular ridges adjacent to less involved areas of the arterial wall. Granuloma pyogenicum is an inflammatory response that can produce a nodular mass, often on the gingiva or the skin. Kaposi sarcoma can produce plaquelike to nodular masses that are composed of irregular vascular spaces lined by atypical-appearing endothelial cells; skin involvement is most common, but visceral organ involvement can occur. Polyarteritis nodosa most often involves small muscular arteries, and sometimes veins. It causes necrosis and microaneurysm formation followed by scarring and vascular occlusion, mainly in the kidney, gastrointestinal tract, and skin of young to middle-aged adults. Takayasu arteritis is seen mainly in children and involves the aorta (particularly the arch) and branches such as the coronary and renal arteries, with granulomatous inflammation, aneurysm formation, and dissection.

A 26-year-old woman has had malaise, arthralgias, and myalgias for the past 2 months. On physical examination, there is no joint swelling or deformity. Laboratory studies show a serum creatinine level of 3.9 mg/dL. A renal biopsy specimen shows a proliferative glomerulonephritis. She receives glucocorticoid therapy for 3 months. She now has left hip pain with movement. On physical examination, there is no swelling or deformity. A radiograph of the left leg and pelvis shows patchy radiolucency and density of the femoral head with flattening of the bone. A total replacement of the left hip is performed, and gross examination of the sectioned femoral head shows collapse of articular cartilage over a pale, wedge-shaped, subchondral area. What is the most likely diagnosis? A Avascular necrosis B Enchondroma C Osteoarthritis D Osteomyelitis E Renal osteodystrophy

A Avascular necrosis of bone (osteonecrosis) represents a localized area of bone infarction, most often in a metaphyseal medullary cavity or subchondral epiphyseal location. The femoral head is most often affected. Underlying conditions associated with osteonecrosis include hemoglobinopathies (sickle cell disease in particular), fracture, barotrauma, hypercoagulable states, and hyperlipidemia. Glucocorticoid therapy decreases osteoblastogenesis to promote avascular necrosis, as in this patient with systemic lupus erythematosus and glomerulonephritis. An enchondroma is a benign tumor of hyaline cartilage that arises in the medullary space of young adults. Osteoarthritis may produce some cartilaginous erosion, but not collapse or bone infarction. Osteomyelitis typically is not so localized, and there is irregular new bone formation (involucrum). The patient's course is quite short for renal osteodystrophy, which is mediated through chronic renal failure and produces lesions such as osteitis fibrosa cystica.

A 22-year-old woman has had a light brown nodule on her left shin for the past year. On examination it is a 0.7-cm firm, nontender nodule. It is excised and on microscopic examination consists of a localized, dermal, spindle cell proliferation with overlying epidermal hyperplasia and downward elongation of hyperpigmented rete ridges. What is the most likely diagnosis? A Dermatofibroma B Leiomyoma C Merkel cell carcinoma D Neurofibroma E Schwannoma

A Benign fibrous histiocytoma, or dermatofibroma, is an indolent lesion that may vary in size over time. It could be a localized response to trauma. The malignant counterpart that grows larger and is more aggressive is the dermatofibrosarcoma protuberans. Leiomyomas, benign smooth muscle neoplasms, can occur in many locations (presumably arising in vascular smooth muscle) but are rare in skin. Merkel cells are neuroendocrine in origin, and a Merkel cell carcinoma resembles a small cell carcinoma of the lung. Neurofibromas can be solitary, or multiple in neurofibromatosis, but they do not have the microscopic epidermal changes seen here. Schwannomas are most likely to arise from a peripheral nerve.

A 5-year-old girl has a routine health checkup. On physical examination, she has scattered 1- to 3-mm, light brown macules on her face, trunk, and extremities. The parents state that these macules become more numerous in the summer months, but fade over the winter. The lesions do not itch, bleed, or hurt. What is the most likely diagnosis? A Freckle B Lentigo C Melasma D Nevus E Vitiligo

A Freckles are common. Individuals with a light complexion and/or red hair are more likely to have freckles. These lesions may be a cosmetic problem, but they have no other significance. A lentigo is a brown macule whose pigmentation is not related to sun exposure. Melasma is most often a masklike area of facial hyperpigmentation associated with pregnancy. Nevi do not wax and wane with sun exposure. Areas of skin with vitiligo are depigmented.

A 75-year-old man has experienced headaches for the past 2 months. On physical examination, his vital signs are temperature, 37° C; pulse, 68/min; respirations, 15/min; and blood pressure, 130/85 mm Hg. His right temporal artery is prominent, palpable, and painful to the touch. His heart rate is regular, and there are no murmurs. His erythrocyte sedimentation rate is 100 mm/hr. A temporal artery biopsy is performed, and the segment of temporal artery excised is grossly thickened and shows focal microscopic granulomatous inflammation. He responds well to corticosteroid therapy. Which of the following complications of this disease is most likely to occur in untreated patients? A Blindness B Gangrene of the toes C Hemoptysis D Malignant hypertension E Renal failure

A Giant cell (temporal) arteritis typically involves large to medium-sized external carotid artery branches in the head (especially temporal arteries), but also vertebral and ophthalmic arteries. Involvement of the latter can affect vision. Because involvement of the kidney, lung, and peripheral arteries of the extremities is much less common, renal failure, hemoptysis, and gangrene of toes are unusual complications of giant cell arteritis. There is no association between hypertension and giant cell arteritis, but some patients may have polymyalgia rheumatica.

A 48-year-old woman had chronic pain of the left shoulder and right hip for 8 months. The pain resolved within 1 month. Two months later, she developed pain in the right knee and ankle, which resolved within 6 weeks. On physical examination, she is now afebrile. There is pain on movement of the left shoulder and right hip. A radiograph of the left arm shows extensive bony erosion of the humeral head. A biopsy specimen of synovium is taken and on microscopic examination shows a marked lymphoplasmacytic infiltrate and arteritis with endothelial proliferation. Which of the following infectious agents is most likely responsible for these findings? A Borrelia burgdorferi B Group B streptococcus C Mycobacterium tuberculosis D Neisseria gonorrhoeae E Treponema pallidum

A Her chronic arthritis is the late, or stage 3, manifestation of Lyme disease, which, as in this case, tends to be remitting and migratory, involving primarily the large joints. The presence of a lymphoplasmacytic infiltrate with endothelial proliferation is characteristic (but not diagnostic) of Lyme arthritis. The infectious agent, Borrelia burgdorferi, is a spirochete that is spread by the deer tick ( Ixodes ). This stage is reached about 2 to 3 years after the initial tick bite, and joint involvement can appear in about 80% of patients. Group B streptococcus may produce an acute osteomyelitis or arthritis in neonates. Tuberculous arthritis may involve large, weight-bearing joints, and it can be progressive, leading to ankylosis. The histologic features of tuberculous arthritis include caseating granulomas. Neisseria gonorrhoeae can cause an acute suppurative arthritis. In both conditions, the inflammatory infiltrate contains a preponderance of neutrophils. Treponema pallidum may produce gummatous necrosis that can involve large joints, and there can be lymphoplasmacytic infiltrates with endarteritis, but syphilitic arthritis is quite rare.

A 10-year-old girl has developed worsening pain in the knees and ankles for the past 3 months and now has difficulty walking. On physical examination, these joints are swollen and warm to the touch, and there is diffuse muscle tenderness. She has a temperature of 39.2° C. There is an erythematous skin rash across the bridge of her nose and on the dorsa of her hands. Hepatosplenomegaly is present, and a friction rub is audible on auscultation of her chest. A joint aspirate is obtained from the left knee, and a microbiological culture of the fluid is negative. On microscopy, the joint fluid has increased numbers of lymphocytes, but few neutrophils. Her condition improves over the next year, and she has no residual joint deformity. Which of the following laboratory findings is most likely to be found in this patient? A ANA serologic titer of 1:1024 B Borrelia burgdorferi serology positive C Chlamydia trachomatis urine culture positive D Ferritin level in serum of 7245 ng/mL E Hemoglobin S on hemoglobin electrophoresis F Rheumatoid factor serologic titer of 1:512 G Serologic test for syphilis (STS) positive H Uric acid level in serum of 15.8 mg/dL

A Juvenile rheumatoid arthritis (RA), or juvenile idiopathic arthritis (JIA), in contrast to the adult type of rheumatoid arthritis, is more likely to be self-limited and nondeforming. JIA typically is rheumatoid factor negative, but ANA positive. JIA is more likely than the adult form to have systemic manifestations, such as rash, myalgia, myocarditis, pericarditis, uveitis, and glomerulonephritis. A positive serologic test for Borrelia burgdorferi is seen in Lyme disease, which tends to be associated with migratory arthritis of large joints. Similar to JIA, Lyme disease produces a chronic deforming arthritis in only about 10% of cases. Chlamydia trachomatis is typically the agent that produces the nongonococcal urethritis seen with reactive arthritis, which, similar to other spondyloarthropathies, most commonly involves the sacroiliac joint. Ferritin levels are markedly increased in hereditary hemochromatosis, in which iron deposition in joints can produce a chronic arthritis similar to osteoarthritis or pseudogout. Sickle cell disease with hemoglobin S can lead to aseptic necrosis, often of the femoral head, and to bone infarcts, with chronic arthritis secondary to bone deformity. Rheumatoid arthritis tends to be recurrent and causes progressive joint deformities, typically of hands and feet. Congenital syphilis can produce periosteitis and osteochondritis with bone deformities; tertiary syphilis in adults can produce gummatous necrosis with joint destruction or loss of sensation, particularly in the lower extremities, leading to repeated trauma that deforms joints (Charcot joint). Some cases of gouty arthritis are accompanied by hyperuricemia; gouty arthritis tends to manifest as an acute attack in older individuals.

A 48-year-old woman has fingers that are tapered and claw-like, with decreased motion at the small joints. There are no wrinkle lines on her facial skin. The microscopic appearance of the skin is shown in the figure. The patient also has diffuse interstitial fibrosis of the lungs, with pulmonary hypertension and cor pulmonale. Which of the following dermal inflammatory cells is the most likely initiator of the process that is the cause of her skin disease? A CD4+ lymphocyte B Macrophage C Mast cell D Neutrophil E Natural killer cell

A She has systemic sclerosis (scleroderma.) The CD4+ lymphocytes are thought to respond to some unknown antigenic stimulation, releasing cytokines that activate further macrophages and mast cells. The result is extensive dermal fibrosis that produces the clinical appearance of sclerodactyly in scleroderma. If she had just skin involvement that progressed late to visceral organs, then she would have limited scleroderma; but if her disease progressed quickly to involve the lungs, then she has diffuse scleroderma. Neutrophils and natural killer cells do not participate in this process. Despite scleroderma being an autoimmune disease, inflammation is minimal. The major finding is progressive fibrosis of skin, lung, and gastrointestinal tract.

A 65-year-old man has developed pruritus followed by blistering skin lesions over the trunk, legs, and arms over the past month. On physical examination, there are 1- to 4-cm tense bullae, particularly over flexural surfaces of skin. A biopsy of one lesion is examined microscopically by direct immunofluorescence staining and shows a subepidermal bulla, with both IgG and C3 deposited linearly along the dermal-epidermal junction. He is treated with topical corticosteroids, and a month later the lesions are healed without scarring. Which of the following components of the skin has most likely been targeted by an autoantibody in this man? A Hemidesmosome B Keratinocyte cell membrane C Lamina densa D Nucleus E Reticulin

A Subepidermal bullae of bullous pemphigoid usually heal without scarring. Subsequent oral lesions may appear. Most often seen in the elderly, this disease results from linear IgG deposition at the basal cell-basement membrane attachment plaques (hemidesmosomes) containing bullous pemphigoid antigen (BPAG). The lamina densa of the basement membrane is not directly involved, and the actual blister of bullous pemphigoid forms in the lamina lucida. In contrast, the antibodies in pemphigus vulgaris attack the desmosomes that attach the epidermal keratinocytes. Antibodies directed against nuclear antigens are more typical for systemic autoimmune diseases such as systemic lupus erythematosus. The antigliadin antibodies of dermatitis herpetiformis cross-react with dermal reticulin, and there are microabscesses at the tips of dermal papillae.

A 45-year-old man has experienced pain in the area of the left hip and upper thigh for the past 7 months. On physical examination, there is tenderness on deep palpation of the left side of the groin. The range of motion at the left hip is decreased, but there is no swelling or warmth on palpation. Pelvic and left leg radiographs show an upper femoral metaphyseal mass lesion that erodes into the surrounding bone cortex. The proximal femur is excised and on sectioning has the gross appearance shown in the figure. Which of the following cell types is most likely to be proliferating in this mass? A Chondrocyte B Osteoblast C Osteoclast D Plasma cell E Primitive neuroectodermal cell

A The glistening, gray-blue appearance shown in the figure is typical of cartilage, and this lesion most likely represents neoplastic proliferation of chondrocytes. This chondrosarcoma has infiltrated the medullary cavity and invaded the overlying cortex, characteristics of a malignant process. Most chondrosarcomas are low grade. They occur in a broad age range, in contrast to many other primary bone tumors that tend to occur in the first two decades of life. Most chondrosarcomas arise toward central portions of the skeleton. Osteoblast proliferation may be seen in a small tumor mass of the cortex known as osteoid osteoma, whereas a larger mass in the axial skeleton may be called an osteoblastoma. Osteosarcomas are derived from osteoblasts, but are malignant. They are usually seen in younger individuals and do not have a bluish-white appearance because they are marked by osteoid production. Giant cell tumors arise during the third to fifth decades; they involve epiphyses and metaphyses. Grossly, they are large, red-brown, cystic tumors. Giant cells resembling osteoclasts are present in giant cell tumors of bone. These tumors are believed to arise from cells of monocyte-macrophage lineage. Atypical plasma cells appear with multiple myeloma. Myelomas are dark red, rounded, lytic lesions that are often multiple. Primitive neuroectodermal cells are present in a Ewing sarcoma.

A 51-year-old man has endured episodes of intense local pain involving his left foot for the past 4 months. The pain may last hours to days. Physical examination identifies the right metatarsophalangeal (MP) joint as the focus of tenderness and swelling, but minimal loss of joint mobility. A painless 2-cm nodule with overlying ulcerated skin is present on the lateral aspect of the MP joint. Beneath the eroded skin is a chalky white deposit of soft material. A firm 1-cm subcutaneous nodule on the extensor surface of the left elbow is excised and has the microscopic appearance shown in the figure. Which of the following mechanisms is most important in causing joint injury in this man? A Activation of neutrophils by phagocytosis of urate crystals B Release of TNF causing acute joint inflammation C Deposition of serum cholesterol into the synovium D Granulomatous inflammation with Mycobacterium tuberculosis infection E Reduced metabolism of homogentisic acid

A The histologic picture is that of a central amorphous aggregate of urate crystals surrounded by reactive fibroblasts and mononuclear inflammatory cells. This is a gouty tophus. Tophi are large collections of monosodium urate crystals that can appear in joints or soft tissues of patients with gout. Large superficial tophi can erode the overlying skin. Precipitation of urate crystals into the joints produces an acute inflammatory reaction in which neutrophils and monocytes can be found. Neutrophils phagocytize urate crystals, which cannot be digested, but cause release of destructive neutrophilic lysosomal enzymes and oxygen free radicals. Release of crystals from the neutrophils perpetuates this cycle of inflammatory response. Inflammation of the joints involves different mechanisms depending on the etiology. In rheumatoid arthritis, release of tumor necrosis factor (TNF) by macrophages plays a central role, as evidenced by the dramatic relief provided by anti-TNF agents. Marked hypercholesterolemia, as occurs in familial hypercholesterolemia, can lead to deposition of cholesterol in tendons and elsewhere. When deposited in tendons, the yellowish lesions are called xanthomas; the cholesterol crystals appear microscopically as clefts in the tissue. Extrapulmonary Mycobacterium tuberculosis infection can cause granulomatous inflammation and chronic arthritis and skin lesions, but there is caseous necrosis with epithelioid cells and no urate crystals. Tuberculous arthritis, in contrast to gouty arthritis, almost never begins in the MP joint. Reduced metabolic breakdown of homogentisic acid occurs in the inborn error of metabolism known as alkaptonuria, and deposition of homogentisic acid (ochronosis) in cartilage causes an arthritis that typically affects large joints, such as knees, intervertebral disks, hips, and shoulders, but small joints of the hands and feet are spared.

A 36-year-old woman has noted a nodule beneath the skin in her left groin since adolescence. On physical examination, the lesion has a 2-cm diameter and is nontender, soft, rubbery, and movable. Which of the following cell types is most likely to comprise this lesion? A Adipocyte B Endothelial cell C Fibroblast D Skeletal muscle E Smooth muscle

A The most common neoplasm is a lipoma, composed of mature adipocytes. They are slow growing and rarely produce problems. Endothelial cells are found in hemangiomas, which are common but firmer. In the groin region, they are likely to be on the skin surface. Fibroblasts may be found in reactive lesions such as fibromatoses, which are firm and immovable. Smooth muscle cells form firm leiomyomas. Neoplasms resembling skeletal muscle (rhabdomyomas) are quite rare.

An 83-year-old woman has pain and limitation of movement affecting the right hip joint, worsening for the past 15 years. Physical examination shows a nodular bony outgrowth can be felt in the distal interphalangeal joint of the right index finger. All other joints are normal. There is no evidence of systemic disease, and cardiovascular and respiratory findings are unremarkable. A radiograph of the affected hip shows narrowing of the joint space and subchondral sclerosis. Laboratory studies do not show rheumatoid factor or antinuclear antibodies. The serum uric acid level is 5 mg/dL. Which of the following factors is most important in the pathogenesis of her disease? A Defects in articular chondrocyte function B Gene mutation involving the synthesis of type I collagen C Infiltration of the synovial membrane by activated CD4+ T cells D Inheritance of HLA-B27 genotype E Partial deficiency of hypoxanthine-guanine phosphoribosyltransferase (HGPRT)

A The progressive involvement of large, weight-bearing joints and osteophytes in the interphalangeal joints in an elderly woman are characteristic of osteoarthritis (OA). The absence of rheumatoid factor, and the asymmetric joint involvement, render the diagnosis of rheumatoid arthritis unlikely. OA is a multifactorial disease in which genetic predisposition and biomechanical forces affect chondrocytes. In early OA, chondrocytes proliferate. This is accompanied by changes in the cartilage matrix owing to secretion of proteases and inflammatory mediators by chondrocytes. Eventually, the ongoing inflammation and injury result in loss of the cartilage that causes reactive subchondral sclerosis. Inherited defects in type I collagen cause a group of rare disorders called osteogenesis imperfecta, which may be lethal in utero or in some cases lead to premature OA. Infiltration of the synovium with CD4+ T cells is seen in rheumatoid arthritis. A partial deficiency of HGPRT gives rise to hyperuricemia and gout. Deficiency of HGPRT is implicated in some cases of gout. HLA-B27 is associated with ankylosing spondylitis and other seronegative spondyloarthropathies.

An epidemiologic study of postmenopausal women is performed. The subjects undergo periodic examination by dual-energy x-ray absorptiometry (DEXA) scan performed on the hip and lumbar vertebrae to evaluate bone mineral density over the next 10 years. They respond to a survey regarding their past and present use of drugs, diet, activity levels, history of bone fractures, and medical conditions. A cohort of the subjects is identified whose bone mineral density is closest to that of the young adult reference range and in whom no bone fractures have occurred. Which of the following strategies is most likely to be supported by the study data to provide the best overall long-term reduction in risk of fracture in postmenopausal women? A Increasing bone mass with exercise during young adulthood B Limited alcohol use, and avoidance of the use of tobacco C Initiation of estrogen replacement therapy after a fracture D Supplementation of the diet with calcium and vitamin D after menopause E Use corticosteroid therapy for inflammatory conditions

A The total bone mass is an important determinant of the subsequent risk of osteoporosis and its complications. A proactive regimen of exercise that puts stress on bones to increase mass before the inevitable loss after age 30 years is most likely to reduce the subsequent risk of osteoporosis. Alcohol and tobacco use are not major risks for osteoporosis. Postmenopausal estrogen or raloxifene therapy can help preserve bone mass; however, by the time a fracture has occurred, there has already been significant bone loss. Likewise, dietary supplements after menopause are not harmful, but at best only partially slow the loss of bone that accompanies aging. Corticosteroid therapy is just one of many risk factors for osteoporosis, but short courses of corticosteroids have minimal effects on bone formation.

A 38-year old healthy man notes occasional pain in his right index finger when using a hammer. On palpation of the right proximal phalanx there is point tenderness. A radiograph shows a 1-cm oval lucency in this phalanx, with a surrounding rim of bright, radiodense bone. The microscopic appearance of the excised lesion is shown in the figure. Which of the following cells is most likely to have given rise to this lesion? A Chondrocyte B Giant cell C Macrophage D Osteoblast E Plasma cell F Fibroblast

A This is an enchondroma. Cartilaginous lesions may be benign when they occur peripherally (hands and feet) and are localized, but a low-grade chondrosarcoma is more likely in a central location. The risk for malignancy is higher with multiple enchondromas (Ollier disease or Maffucci syndrome with mutations in isocitrate dehydrogenase genes). Giant cells are seen in many mass lesions of bone, but particularly in giant cell tumors and aneurysmal bone cysts occurring in larger bones. Macrophages may increase in lysosomal storage diseases such as Gaucher disease that involve bone marrow. Osteoblasts may be seen in an osteoid osteoma, which is more likely to occur at a younger age and produce pain at night. Plasma cells in myeloma are unlikely to occur at this age or at this site. Fibroblasts may be seen in localized fibrous lesions such as fibrous cortical defects that involve long bones.

A 74-year-old woman has noted increasing size and number of darker brown patches on the dorsum of each hand for the past 15 years. They do not change with sun exposure, are nonpruritic, and nontender. On examination, these 0.5- to 1-cm lightly pigmented lesions are flat. Which of the following is the most likely microscopic finding in these lesions? A Basal melanocytic hyperplasia B Dermal nevus cells C Loss of melanin in surrounding skin D Mast cell proliferation E Pigmented fungal hyphae

A Unlike a freckle that darkens upon light exposure, lentigo (pleural: lentigenes) does not. There is focal increase in melanocytes of the basal layer, along with thinning of epidermis and elongation of rete ridges. Lentigenes can occur at any age; those in older persons may be termed senile lentigenes, or age spots. A nevus typically forms a nodule and likely appears much earlier in life. Loss of melanin is termed vitiligo, and is a separate process from melanocyte hyperplasia. A collection of mast cells can produce urticaria pigmentosa, which often has a light brown appearance, but pruritus is common. Some superficial fungal infections (black piedra) may be pigmented, but they typically do not persist for years.

A 28-year-old man flips over an all-terrain vehicle, and he lands on his leg. On physical examination there is intense pain on palpation over the right shin, but there is no shortening of the limb. The overlying skin is intact. A radiograph shows right tibial and fibular midshaft fracture into multiple bone fragments. Which of the following terms best describes these fractures? A Comminuted B Compound C Displaced D Incomplete E Pathologic

A When the bone is splintered into fragments, the fracture is comminuted, and will likely require orthopedic appliances to align fragments for healing. A compound fracture penetrates the skin. Displaced fractures have nonaligned bone, and some deformity may result. The bone remains contiguous with an incomplete fracture that does not extend completely across the shaft of the bone. A pathologic fracture occurs in a location weakened by a preexisting disease, such as a metastatic lesion.

A 17-year-old girl has hundreds of skin lesions on her body that have been forming since childhood. On physical examination, 0.4- to 1.7-cm, macular to slightly raised, plaquelike, dark brown pigmented lesions are present on sun-exposed and non-sun-exposed areas of skin. The lesions have irregular contours, and there is variability in pigmentation. She says that her 15-year-old brother has similar lesions. She is most likely to have an inherited mutation involving which of the following genes? A CDKN2A B FGFR3 C NF1 D PTCH E TSC1

A - The clinical appearance, distribution, and occurrence in two siblings suggest that these lesions represent the dysplastic nevus syndrome. Dysplastic nevi are precursors of malignant melanoma. The most important gene in familial cases is that of cyclin-dependent kinase inhibitor 2 (CDKN2A) , which encodes several tumor suppressors including p16/INK4A. FGFR3 mutations may be found with seborrheic keratoses in older adults. The PTCH gene is implicated in the pathogenesis of sporadic basal cell carcinomas and the familial nevoid basal cell carcinoma syndrome. TSC1 mutations are associated with tuberous sclerosis.

A 47-year-old man has had dull, constant pain in the midsection of the right thigh for the past 4 months. On physical examination, there is pain on palpation of the anterior right thigh, which worsens with movement. The right thigh appears to have a larger circumference than the left thigh. A radiograph of the right upper leg and pelvis shows no fracture, but there is an ill-defined soft-tissue mass anterior to the femur. MRI shows a 10 × 8 × 7 cm solid mass deep to the quadriceps, but it does not involve the femur. Karyotypic analysis of tumor cells reveals t(12;16)(q13;p11) with amplification of MDM2 gene. What is the most likely diagnosis? A Chondrosarcoma B Liposarcoma C Metastatic adenocarcinoma D Nodular fasciitis E Osteosarcoma F Rhabdomyosarcoma

B A large, deep soft-tissue mass suggests cancer, most likely a sarcoma. Liposarcomas are located in deep soft tissues, can be indolent, and can reach a large size. They are the most common sarcomas of adulthood. Chondrosarcomas can be seen over a wide age range, but they arise within bone. Carcinomas are far more common in adults than sarcomas, and can metastasize, but such a large lesion in soft tissue is unlikely to be a metastasis. Nodular fasciitis is a reactive fibroblastic lesion of young adults, usually on the upper extremities and trunk, and can develop several weeks after local trauma. Osteosarcomas generally occur in individuals younger than 20 years and typically arise in the metaphyseal region of long bones. Rhabdomyosarcoma occurs in children and is most often a tumor of the head and neck, genitourinary tract, or retroperitoneum.

A 64-year-old man has noted changes in the texture and color of skin in his armpits and groin over the past 3 months. On physical examination, there is thickened, darkly pigmented skin in the axillae and flexural areas of the neck and groin. These areas are neither painful nor pruritic. A punch biopsy specimen of axillary skin shows undulating epidermal acanthosis with hyperkeratosis and basal layer hyperpigmentation. These lesions are most likely to be cutaneous markers of which of the following underlying diseases? A AIDS B Colonic adenocarcinoma C Langerhans cell histiocytosis D Mastocytosis E Systemic lupus erythematosus

B Acanthosis nigricans may be a cutaneous marker for benign and malignant neoplasms. The skin lesions often precede signs and symptoms of associated cancers. They are believed to arise from the action of epidermal growth-promoting factors produced by neoplasms. Various skin lesions are associated with AIDS, including disseminated infections and papulosquamous dermatoses, although not pigmented lesions. Involvement of the skin in Langerhans cell histiocytoses typically occurs in children; they produce reddish papules or nodules or erythematous scaling plaques because the Langerhans cell infiltrates in the dermis. Skin lesions of mastocytosis in adults often exhibit urticaria. The erythematous rashes that develop in systemic lupus erythematosus are the result of antigen-antibody complex deposition and often exhibit photosensitivity.

A 75-year-old woman has experienced increasing dull but constant pain in the back, right chest, left shoulder, and left upper thigh for the past 6 months. She has now developed a sudden, severe, sharp pain in the left thigh. On physical examination, she has intense pain on palpation of the upper thigh, and the left leg is shorter than the right. A radiograph of the left leg shows a fracture through the upper diaphyseal region of the femur in a 5-cm lytic area that extends through the entire thickness of the bone. A bone scan shows multiple areas of increased uptake in the left femur, pelvis, vertebrae, right third and fourth ribs, upper left humerus, and left scapula. Laboratory studies show serum creatinine, 0.9 mg/dL; total protein, 6.7 g/dL; albumin, 4.5 g/dL; total bilirubin, 1 mg/dL; AST, 28 U/L; ALT, 22 U/L; and alkaline phosphatase, 202 U/L. What is the most likely diagnosis? A Hyperparathyroidism B Metastatic carcinoma C Multiple myeloma D Osteochondromatosis E Paget disease of bone F Polyostotic fibrous dysplasia

B An elevated alkaline phosphatase level in an older adult should raise the suspicion of bone metastases, particularly when there is a "pathologic" fracture resulting from a bone lesion, rather than a fracture from trauma. Likely primary sites include the breast (in women), prostate (in men), lung (in smokers), kidney, and thyroid. Hyperparathyroidism can lead to osteitis fibrosa cystica with lytic lesions that are usually small, involve just cortex, and appear first in phalanges. Multiple myeloma can produce lytic bone lesions, but the patient's serum gamma globulin level is not elevated. Osteochondromas are exostoses and do not produce lytic bone lesions. Paget disease of bone is characterized by osteolysis coupled with bone formation, but without lytic lesions. Fibrous dysplasia coupled with café-au-lait spots on skin and with endocrinopathies is known as McCune-Albright syndrome; this is a rare condition that occurs in young girls.

A 76-year-old man has had a slowly enlarging nodule on his right eyelid for 4 years. On physical examination, there is a 0.3-cm pearly nodule on the upper eyelid near the lateral limbus of the right eye. The lesion is excised, but multiple frozen sections are made during the surgery to minimize the extent of the resection and preserve the eyelid. The microscopic appearance of the lesion is shown at low magnification in the figure. What is the most likely diagnosis? A Actinic keratosis B Basal cell carcinoma C Dermatofibroma D Malignant melanoma E Nevocellular nevus

B Basal cell carcinomas arise as pearly papules on sun-exposed areas of the skin, particularly the face. They slowly infiltrate surrounding tissues, gradually enlarging. Although it rarely metastasizes, a basal cell carcinoma can have serious local effects, particularly in the area of the eye. An actinic keratosis is a premalignant lesion of the epidermis that does not invade surrounding tissue. A dermatofibroma is a benign lesion of the dermis composed of cells resembling fibroblasts. Melanomas are usually pigmented, and they are composed of polygonal or spindle cells that tend to grow in sheets and infiltrate to produce a grossly irregular border to the lesion. A nevus is a small, localized, benign lesion.

A 46-year-old man has had a slowly enlarging nodule on his trunk for the past 10 months. On examination, the 5-cm lesion is in the right anterior axillary line at the level of the nipple and has a central ulcerated plaque. The lesion is widely excised and on microscopic examination is composed of dense spindle cells in a storiform pattern that invades the dermal adipose tissue. The lesion recurs within a year, and the man is treated with a tyrosine kinase inhibitor. Molecular analysis of the lesion is most likely to show activation with overexpression of which of the following genes? A TSC1 B PDGFB C FGFR3 D KIT E PTCH1

B Dermatofibrosarcoma protuberans is the malignant counterpart of dermatofibroma. The PDGFB gene is juxtaposed with the promoter region of COL1A1, leading to up-regulation of a growth-promoting factor. Imatinib mesylate can be employed to inhibit the PDGF receptor tyrosine kinase to control lesions that are recurrent or metastatic. FGFR3 mutations are more characteristic for seborrheic keratoses. KIT mutations are found in mast cell proliferations. Basal cell carcinomas have PTCH1 mutations. TSC1 mutations are found in tuberous sclerosis, with skin manifestations, including ash-leaf patches, shagreen patches, subungual fibromas, and angiofibromas.

A 43-year-old woman has experienced increasing difficulty in swallowing over the past year. She also has experienced diarrhea with a 5-kg weight loss in the past 6 months. She reports increasing dyspnea during this time. On physical examination, crackles are auscultated in all lung fields, but heart sounds are faint. Echocardiography shows a large pericardial effusion. The ANA test result is positive at 1:512 with a nucleolar pattern. The anti-DNA topoisomerase antibody titer is 1:1024. Which of the following serious complications is most likely to occur in patients with this disease? A Hepatic failure B Malignant hypertension C Meningitis D Perforated duodenal ulcer E Squamous cell carcinoma

B Diffuse systemic sclerosis (scleroderma) initially has widespread skin involvement but rapidly progresses to visceral organ involvement, including the small arteries of the kidney. These are damaged by a hyperplastic arteriolosclerosis that can be complicated by very high blood pressure and renal failure. Meningitis and liver failure are not typical features of autoimmune diseases. With scleroderma, the gastrointestinal tract undergoes fibrosis with obstruction and malabsorption, without any tendency to perforation or ulceration. Risk factors for esophageal squamous carcinoma include smoking and alcohol abuse.

An 8-year-old child has had abdominal pain and dark urine for 10 days. Physical examination shows blotchy purple skin lesions on the trunk and extremities. Urinalysis shows hematuria and proteinuria. Serologic test results are negative for MPO-ANCA (P-ANCA) and PR3-ANCA (C-ANCA). A skin biopsy specimen shows necrotizing vasculitis of small dermal vessels. A renal biopsy specimen shows immune complex deposition in glomeruli, with some IgA-rich immune complexes. Which of the following is the most likely diagnosis? A Giant cell arteritis B Henoch-Schönlein purpura C Polyarteritis nodosa D Takayasu arteritis E Telangiectasias

B In children, Henoch-Schönlein purpura is the multisystemic counterpart of the IgA nephropathy seen in adults. The immune complexes formed with IgA produce the vasculitis that affects mainly arterioles, capillaries, and venules in skin, gastrointestinal tract, and kidney. In older adults, giant cell arteritis is seen in external carotid branches, principally the temporal artery unilaterally. Polyarteritis nodosa is seen most often in small muscular arteries and sometimes veins, with necrosis and microaneurysm formation followed by scarring and vascular occlusion. This occurs mainly in the kidney, gastrointestinal tract, and skin of young to middle-aged adults. Takayasu arteritis is seen mainly in children and involves the aorta (particularly the arch) and branches such as coronary and renal arteries, with granulomatous inflammation, aneurysm formation, and dissection. Telangiectasias are small vascular arborizations seen on skin or mucosal surfaces.

A 50-year-old man with muscle pain and fever for a month now notes darker colored urine for the past 2 weeks. On physical examination he has palpable purpuric lesions of his skin. Urinalysis shows hematuria and proteinuria. Serum laboratory findings include a mixed cryoglobulinemia with a polyclonal increase in IgG, as well as a high titer of anti-neutrophil cytoplasmic autoantibodies, mainly antimyeloperoxidase (MPO-ANCA, or P-ANCA). A skin biopsy is performed. What pathologic finding is most likely to be observed in this biopsy? A Giant cells and macrophages B Medial fibrinoid necrosis C Microabscesses D Mycotic aneurysms E Perivascular eosinophilic infiltrates

B Microscopic polyangiitis involves small vessels, typically capillaries. Kidneys and lungs are commonly involved, but many organs can be affected. There may be an underlying immune disease, chronic infection, or drug reaction. Giant cell arteritis typically involves arterial branches of the external carotid, most often the temporal artery. Microabscesses may be present with an infectious process, or with thromboangiitis obliterans (Buerger disease), which typically involves lower extremities. Mycotic aneurysms occur when a focus of infection, often from a septic embolus, weakens an arterial wall so that it bulges out. Perivascular eosinophilic infiltrates may be seen with Churg-Strauss syndrome, which typically involves the lungs.

A 29-year-old woman has had increasing weakness over the past year, and now has difficulty climbing a single flight of stairs. Her muscles are sore most of the time. She has little difficulty writing or typing, however. During the past 3 months, she has had increasing difficulty swallowing. She has experienced chest pain for the past week. On physical examination, she is afebrile. Her blood pressure is 115/75 mm Hg. Muscle strength is 4/5 in all extremities. No rashes are present. She has 2+ pitting edema to the knees. Rales are auscultated over lower lung fields. Laboratory studies show serum creatine kinase level of 458 U/L and Jo-1 antibodies. Which of the following additional complications of her disease is she most likely to have? A Bony ankylosis B Myocarditis C Pericarditis D Sclerodactyly E Urethritis F Xerophthalmia

B Muscle weakness in polymyositis tends to be symmetric, and proximal muscles are involved first. This condition differs from dermatomyositis in that there is no skin involvement, and polymyositis typically affects adults. On biopsy, the skeletal muscle shows infiltration by lymphocytes along with degeneration and regeneration of muscle fibers. The lymphocytes are cytotoxic CD8+ cells. Some patients may have myocarditis, vasculitis, or pneumonitis, but in contrast to dermatomyositis, the risk of cancer is equivocal. Bony ankylosis is a feature of progressive or recurrent joint inflammation with rheumatoid arthritis. Pericarditis is most likely to be a feature of systemic lupus erythematosus or diffuse systemic sclerosis. Sclerodactyly is a feature of scleroderma. When not extensive, it typically indicates limited scleroderma (CREST syndrome); when extensive, it indicates diffuse scleroderma, which has a poorer prognosis. Nongonococcal urethritis, conjunctivitis, and arthritis are seen with reactive arthritis. Xerophthalmia (usually with accompanying xerostomia) is seen in Sjögren syndrome.

A 26-year-old man is struck in the left arm by a swinging steel beam at a construction site. On physical examination, a 4-cm area of the lateral upper left arm exhibits swelling and redness with pain on palpation. A radiograph of the left arm shows no fracture. Three weeks later, there now is a 2-cm painful, well-circumscribed, subcutaneous mass at the site of the original injury. A radiograph shows a solid soft-tissue mass. Which of the following lesions is most likely to be present in this man? A Lipoma B Nodular fasciitis C Organizing abscess D Pleomorphic fibroblastic sarcoma E Superficial fibromatosis

B Nodular fasciitis is a reactive fibroblastic proliferation that is seen in the upper extremities and trunk of young adults, sometimes occurring after trauma. A lipoma is a common benign soft-tissue tumor that is not painful and does not follow trauma. A contusion is unlikely to lead to abscess formation because there is no disruption of the skin to allow entry of infectious agents. Pleomorphic fibroblastic sarcoma is most likely to arise in the retroperitoneum and deep soft tissues of extremities in older adults. A superficial fibromatosis is a deforming lesion of fascial planes that develops over a long period, and the most common is a Dupuytren contracture involving the palm of the hand.

A 54-year-old woman has been bothered by a chronic, dry cough for the past 5 years. She has had increasing difficulty with blurred vision for the past year. On physical examination, she has a perforated nasal septum, bilateral mild corneal scarring, and fissuring of the tongue and corners of her mouth. Laboratory studies show antibodies to SS-A and SS-B. Her serum creatinine is 2.5 mg/dL and urea nitrogen 25 mg/dL. A renal biopsy specimen examined microscopically shows tubulointerstitial nephritis but no glomerular involvement. Which of the following is the most serious condition likely to occur in this patient? A Endocarditis B Non-Hodgkin lymphoma C Renal failure D Salivary gland cancer E Esophageal dysmotility F Urethritis

B Sjögren syndrome is characterized by immunologically mediated destruction of salivary and lacrimal glands and other exocrine glands lining the respiratory and gastrointestinal tracts. Dryness and crusting of the nose can lead to perforation of the nasal septum. In 25% of cases, extraglandular tissues, such as lung, skin, kidney, and muscles, may be involved. The immune dysregulation that accompanies autoimmune diseases increases the risk for B cell lymphoid malignancies, such as MALT lymphoma. Libman-Sacks endocarditis is most often a feature of systemic lupus erythematosus (SLE). Renal failure is more likely to occur with SLE from glomerulonephritis. Esophageal dysmotility is a feature of scleroderma. When not extensive, it typically indicates limited scleroderma (CREST syndrome); when extensive, it indicates diffuse scleroderma, which has a poorer prognosis. Salivary gland cancers are unlikely to be associated with autoimmune diseases. Nongonococcal urethritis is seen in reactive arthritis, along with conjunctivitis and arthritis.

A 13-year-old, previously healthy boy has had pain in the right leg for the past month. There is no history of trauma or recent illness. On physical examination, there is warmth and tenderness to palpation of the right lower thigh anteriorly, and the circumference of the right thigh is slightly larger than that of the left. His temperature is 39° C. A radiograph of the right leg shows a 6-cm expansile mass in the diaphyseal region of the right lower femur that extends into the soft tissue and is covered by layers of reactive bone. A biopsy of the mass is done, and microscopic examination shows sheets of closely packed primitive cells with small, uniform nuclei and only scant cytoplasm. Karyotypic analysis of the tumor cells shows a t(11;22) translocation. What is the most likely diagnosis? A Chondrosarcoma B Ewing sarcoma C Giant cell tumor D Metastatic carcinoma E Osteosarcoma F Plasmacytoma

B The histologic appearance is characteristic of a Ewing sarcoma. The radiologic appearance of the mass in this child is typical for a malignant tumor, with bone destruction and soft-tissue extension. The two most common malignant bone tumors in children are osteosarcoma and Ewing sarcoma. Osteosarcomas typically arise in the metaphyseal region, whereas Ewing sarcoma arises in the diaphyseal region of long tubular bones, as seen in this case. This tumor usually occurs in patients 10 to 15 years old. The t(11;22) translocation is present in about 85% of Ewing sarcomas and the related primitive neuroectodermal tumors (PNETs), which belong to the small round cell tumors of childhood that can be difficult to distinguish microscopically. The translocation gives rise to the EWS-FLI1 fusion gene, now considered the definitive test for the diagnosis of these tumors. Ewing sarcomas often produce tender masses with fever and leukocytosis, mimicking acute osteomyelitis. A chondrosarcoma can occur across a wide age range, in contrast to most primary malignancies arising in bone, which occur most often in the first two decades; most are sufficiently differentiated so that a cartilaginous matrix is apparent on microscopic examination. A giant cell tumor is a benign but locally aggressive lesion that arises in the epiphysis of the long bones of young adults and has a "soap bubble" radiographic appearance. Metastatic carcinoma is the most common tumor of adults involving bone because there are far more carcinomas than primary bone malignancies; childhood bone metastases are rare. An osteosarcoma typically arises in the metaphyseal region, and the malignant spindle cells produce an osteoid matrix. A plasmacytoma produces a focal lytic lesion within bone, and microscopically there are recognizable plasma cells.

A 57-year-old woman has noticed increasing deformity and difficulty with movement involving her left hand over the past 6 months. On physical examination, there is a contracture involving the third digit of her left hand that prevents her from fully extending this finger. A firm, hard, cordlike 1 × 3 cm area is palpable beneath the skin of the left palm. Microscopically, which of the following is most likely to be seen in greatest abundance composing this lesion? A Atypical spindle cells B Collagen C Dystrophic calcification D Granulation tissue E Lipoblasts

B The patient has superficial fibromatosis that has produced a lesion best known as a Dupuytren contracture. These lesions contain mature fibroblasts surrounded by dense collagen. A hard, firm lesion of this size is unlikely to be malignant. Though some fibrobastic cells are present with a spindle shape, they are not numerous and generally not significantly atypical. Dystrophic calcification occurs in necrotic tissues; it is not commonly a localized mass. Granulation tissue from an injury would give rise to a stable scar without such severe retraction. Lipoblasts are seen in a liposarcoma, which is more likely to arise in deep soft tissues, such as thigh or retroperitoneum.

A 39-year-old man has experienced back pain for 3 months. He has had a chronic cough for 2 years. On physical examination, there is tenderness to palpation over the lumbar vertebrae, but no warmth, swelling, or erythema. A radiograph of the spine shows a compressed fracture at the L2 level. CT scan of the abdomen shows an abscess involving the right psoas muscle. Infection with which of the following microbial agents is most likely to produce these findings? A Cryptococcus neoformans B Mycobacterium tuberculosis C Shigella flexneri D Staphylococcus aureus E Streptococcus pyogenes F Treponema pallidum

B The presence of a destructive lesion in the vertebrae with extension of the disease along the psoas muscle, without signs of acute inflammation (cold abscess) is characteristic of tuberculous osteomyelitis. Tuberculosis of bones usually results from hematogenous spread of an infection in the lung. Long bones and vertebrae are the favored sites for tuberculosis involving the skeletal system. Dissemination of Cryptococcus neoformans infection from the lungs occurs most commonly in immunocompromised patients, but infrequently produces osteomyelitis. Shigella species are unlikely to cause osteomyelitis but may be associated with reactive arthritis. Staphylococcal osteomyelitis is more common than streptococcal osteomyelitis, but both organisms are more likely to involve long bones and produce an acute hot joint. Treponema pallidum may produce gummatous necrosis in the tertiary stage, but this involves soft tissues more than bone.

A 7-year-old boy sustained an open compound fracture of the right tibia and fibula in a fall from a barn loft to the floor below. On physical examination, the lower tibia and fibula can be seen protruding from the lower leg. The fracture is set by external manipulation, and the skin wound is sutured, but nothing more is done. One year later, he continues to have pain in the right leg, and a draining sinus tract has developed in the lateral lower right leg. A radiograph of the lower right leg is now most likely to show which of the following? A Cortical nidus with surrounding sclerosis B Involucrum and sequestrum C Osteolysis with osteosclerosis D Soft-tissue hemorrhage and swelling E Tumor mass with bony destruction

B This patient has chronic osteomyelitis. The most likely sequence of events is the occurrence of a compound fracture that became infected by direct extension of bacteria into the bone. The subsequent care for this patient was inadequate, and he developed chronic osteomyelitis. Infection of the bone and the associated vascular compromise caused bone necrosis, giving rise to a dead portion of bone, called sequestrum. With chronicity, a shell of reactive new bone, called involucrum, is formed around the dead bone. A nidus with surrounding sclerosis suggests an osteoid osteoma. Osteolysis and osteosclerosis are features of bone remodeling with Paget disease of bone. Soft-tissue hemorrhage and swelling should be minimal and resolve soon after the fracture is stabilized. A mass suggests a malignancy, and the most common neoplasm to develop in a sinus tract draining from osteomyelitis is squamous cell carcinoma, but this is uncommon.

A 44-year-old woman has had tightening of the skin of her fingers for the past 2 years, making them difficult to bend. She has had increasing difficulty swallowing for the past 8 months. During the past winter, her fingers became cyanotic and painful on exposure to cold. On physical examination, the skin on her face, neck, hands, and forearms appears firm and shiny. Her blood pressure is 200/130 mm Hg. A chest radiograph shows prominent interstitial markings, and lung function tests indicate moderately severe restrictive pulmonary disease. The result of her DNA topoisomerase I antibody test is positive. Which of the following conditions is she most likely to have? A Ankylosing spondylitis B Diffuse scleroderma C Discoid lupus erythematosus D Limited scleroderma E Rheumatoid arthritis F Systemic lupus erythematosus

B This patient has cutaneous and visceral manifestations of diffuse systemic sclerosis (diffuse scleroderma). Raynaud phenomenon, skin changes, and esophageal dysmotility also can occur in limited scleroderma (the former CREST syndrome), but lung and renal involvement typically do not. In diffuse systemic sclerosis, the anti-DNA topoisomerase I antibody is often present, and patients can develop interstitial lung disease and renal disease with hyperplastic arteriolosclerosis. A feature of discoid lupus erythematosus is skin rashes, but usually there is no internal organ involvement. Ankylosing spondylitis is one of the spondyloarthropathies; it is characterized by low back pain from sacroiliitis and positive serology for HLA-B27. In rheumatoid arthritis, there is often progressive joint deformity; the serologic tests likely to be positive include rheumatoid factor and antibodies to cyclic citrullinated peptide (anti-CCP). The anti-Smith or anti-double-stranded DNA antibodies are more specific for systemic lupus erythematosus, and renal disease in these patients is most likely due to glomerulonephritis.

A 48-year-old man has had increasing pain in the left knee for the past 4 years, but the pain has become worse in the past week. On physical examination, the left knee is slightly swollen and warm to the touch. The cell count of a joint aspirate shows increased neutrophils. The figure shows a smear preparation of the fluid examined with polarized light microscopy ( arrow in axis of red compensator). The patient experiences reduced knee joint mobility over the next 5 years. He also develops congestive heart failure, diabetes mellitus, and hepatic cirrhosis. Which of the following laboratory findings is most characteristic of this disease process? A ANA serologic titer of 1:1024 B Ferritin level in serum of 7245 ng/mL C Rheumatoid factor serologic titer of 1:512 D Serologic test for syphilis (STS) positive E Uric acid level in serum of 15.8 mg/dL

B This patient shows evidence of hemochromatosis-caused skin pigmentation, heart failure, diabetes, and cirrhosis. The figure shows negatively birefringent (blue) rhomboidal calcium pyrophosphate crystals that have been deposited in the articular matrix. In progression of the disease, the crystals can seed the joint space and give rise to pseudogout, or calcium pyrophosphate dihydrate deposition (CPDD) disease, also called chondrocalcinosis. CPDD can be primary (hereditary) or, more commonly, secondary to various systemic diseases, such as hemochromatosis or, in the elderly, secondary to preexisting joint damage from other conditions. In most autoimmune diseases with a positive ANA result, such as systemic lupus erythematosus, there are arthralgias, but no arthritis, and little or no joint swelling, destruction, or deformity occurs. Rheumatoid arthritis tends to be recurrent and causes progressive joint deformities, typically of the hands and feet. Congenital syphilis can produce periosteitis and osteochondritis with bone deformities; tertiary syphilis in adults can produce gummatous necrosis with joint destruction or loss of sensation, particularly in the lower extremities, leading to repeated trauma that deforms joints (Charcot joint). Some cases of gouty arthritis are accompanied by hyperuricemia; gouty arthritis tends to manifest as acute attacks in older individuals.

A 14-year-old girl experiences severe pain in the right leg after performing a gymnastic floor exercise. On physical examination, there is marked pain on palpation of the right lower thigh just above the knee. Radiographs show a pathologic fracture across a 3-cm lower femoral diaphyseal lesion that has central lucency with a thin sclerotic rim. The lesion is completely intramedullary and well circumscribed. A bone biopsy specimen of the affected region is taken and microscopically shows scattered trabeculae of woven bone in a background of fibroblastic proliferation. What is the most likely diagnosis? A Ewing sarcoma B Fibrous dysplasia C Fracture callus D Osteogenic sarcoma E Osteoid osteoma

B This single focus (monostotic) fibrous dysplasia weakens the bone to the point of pathologic fracture. This benign tumorlike condition is uncommon. The histologic appearance of woven bone in the middle of benign-looking fibroblasts is characteristic. Seventy percent of cases are monostotic, and the ribs, femur, tibia, mandible, and calvaria are the most frequent sites of involvement. Local deformity and, occasionally, fracture can occur. Polyostotic fibrous dysplasia may involve craniofacial, pelvic, and shoulder girdle regions, leading to severe deformity and risk for fracture. Ewing sarcoma usually occurs in the diaphyseal region of the long bones and is identified histologically by sheets of small, round cells. A fracture callus should not be so localized within bone, and should not develop so quickly following trauma. An osteosarcoma is typically a large destructive lesion without central lucency. An osteoid osteoma has a small central nidus with surrounding sclerosis.

A 24-year-old woman has had increasing malaise; facial skin lesions shown in the figure are exacerbated by sunlight exposure; and arthralgias and myalgias for the past month. On physical examination she has mild pedal edema. On auscultation, a friction rub is audible over the chest. Laboratory findings include pancytopenia and serum creatinine 3 mg/dL. Urinalysis shows hematuria and proteinuria. A serologic test for syphilis yields a false-positive result. A renal biopsy shows granular deposits of IgG and complement in the mesangium and along the basement membrane. Which of the following mechanisms is most likely involved in the pathogenesis of her disease? A Antiphospholipid antibodies B Defective clearance of apoptotic nuclei C Increased production of IFN-γ D Molecular mimicry E Superantigen activation of T cells

B This young woman has a classic picture of systemic lupus erythematosus (SLE)—the erythematous malar facial skin rash shown in the figure, and renal failure with proteinuria and hematuria from immune complex deposition in the glomeruli. Defective clearance and hence increased burden of nuclear apoptotic bodies in thymic lymphocyte development is considered a fundamental mechanism that underlies SLE. This along with loss of self-tolerance to nuclear antigens gives rise to the pathogenic DNA-anti DNA immune complexes, as measured by the antinuclear antibody test. Antiphospholipid antibodies may be present with SLE, but lead to coagulopathy. IFN-γ is a product of CD4+ T cells and NK cells. There is no evidence of delayed hypersensitivity or NK cell dysfunction in SLE. Molecular mimicry occurs when a microbial antigen cross-reacts with a normal tissue as in rheumatic fever. Widespread and nonspecific activation of T cells by superantigens occurs in toxic shock syndrome.

A 13-year-old girl has two nontender lesions on her fingers that have appeared over the past 5 months. On physical examination, there are 0.5-cm slightly raised, pebbly-surfaced, gray-white papules, one on the dorsum of her distal right index finger and another periungual to her little finger. They gradually disappear over the next 18 months. Which of the following is the most likely factor in the pathogenesis of her lesions? A BRAF gene mutation B Human papillomavirus infection C IgA antibody deposition D Type IV hypersensitivity reaction E Vitamin B 3 deficiency

B Verrucae, or warts, are quite common, particularly in children, and tend to persist for up to a couple of years. There are subtypes of human papillomavirus (HPV) associated with distinct clinical appearances: verruca vulgaris on the hands, verruca plana (flat wart) on the face and hands, verruca palmaris on the palms, verruca plantaris on the soles, and condyloma acuminatum (venereal wart) on genitalia. BRAF mutations can be present with melanocytic proliferations, such as dysplastic nevi. IgA deposition is found with dermatitis herpetiformis. Contact dermatitis is a form of type IV hypersensitivity. A photosensitive dermatitis may be seen with niacin deficiency.

A 60-year-old woman exposed to tuberculosis is found to have a positive tuberculin skin test. She receives prophylactic therapy that includes isoniazid. She develops arthralgias, myalgias, and a malar erythematous rash 9 months later. Laboratory findings include an ANA titer of 1:2560 in a diffuse pattern. Anti-double-stranded DNA antibodies are not present. Which of the following autoantibodies has the greatest specificity for her condition? A Anti-Sm B Anticentromere C Antihistone D Anti-Jo-1 E Anti-U1-ribonucleoprotein F Anti-SS-A

C A drug-induced systemic lupus erythematosus (SLE)-like condition may be caused by drugs such as isoniazid, procainamide, and hydralazine. Test results for ANA are often positive, but test results for anti-double-stranded DNA are negative. Antihistone antibodies are present in many cases. Characteristic signs and symptoms of SLE may be lacking, and renal involvement is uncommon. Remission occurs when the patient stops taking the drug. Anti-Sm antibody shows specificity for SLE. Anti-Jo-1 antibody has specificity for polymyositis/dermatomyositis. Anti-U1-ribonucleotide protein has specificity for mixed connective tissue disease. Anticentromeric antibody is most likely to be present with limited scleroderma (CREST syndrome). Anti-SS-A antibody is most characteristic of Sjögren syndrome.

A 47-year-old woman has had increasing pain and deformities in her hands for the past 10 years. On physical examination, she has metacarpophalangeal joint swelling, erythema, and tenderness. There is a subcutaneous nodule on the ulnar aspect of the right forearm. A biopsy specimen of the nodule has the microscopic appearance depicted in the figure. Which of the following therapies is most likely to be effective in this patient? A Bisphosphonates B Broad-spectrum antibiotics C Anti-TNF agents D Uricosuric agent cell cycle inhibitors

C A rheumatoid nodule is shown in the figure. Subcutaneous rheumatoid nodules such as this are typically found over extensor surfaces. Features of chronic rheumatoid arthritis (RA) include bilateral symmetric involvement of joints, destruction of joints with characteristic deformities, and presence of rheumatoid nodules. Although the pathogenesis of RA is complex, it is believed that the tissue injury is mediated by an autoimmune reaction in which CD4+ T cells secrete cytokines that have a cascade of effects on B cells, macrophages, and endothelial cells. B cells are driven to form rheumatoid factors, which form immune complexes in the joint; macrophages secrete cytokines such as tumor necrosis factor (TNF) and interleukin-1, which activate cartilage cells, fibroblasts, and synovial cells; and endothelial cell activation promotes accumulation of inflammatory cells in the synovium. Together, these processes form a pannus and eventually cause joint destruction. The central role of TNF in orchestrating joint destruction is the basis for the highly successful treatment of RA with anti-TNF therapy. Bisphosphonates diminish osteoclast activity to treat conditions with bone loss. RA is noninfectious, so antibiotics are not indicated. Cell cycle inhibitors as part of chemotherapy are reserved for malignancies; RA does not give rise to cancer. Although a gouty tophus can occur in soft tissue and resemble the lesion shown, the distribution of lesions does not fit with gout.

A 25-year-old man has had right knee pain with "popping and catching" for the past 2 years. On examination there is reduced range of motion at the right knee, but no tenderness or swelling. MR imaging reveals a 3-cm mass in the anterior joint space. Arthroscopic surgery with partial synovectomy is performed, and a nodular, encapsulated mass is removed. Microscopic examination of the mass shows synoviocyte-like tumor cells in a hyalinized stroma containing osteoclast-like giant cells and hemosiderin. An antagonist to which of the following growth factors is most useful in treating this lesion? A EGF B FGF C M-CSF D PDGF E TGF-β F VEGF

C A tenosynovial giant cell tumor (formerly called pigmented villonodular synovitis ) is the most common soft tissue tumor of the hand, but can occur around the knee as either a localized or diffuse proliferation. An acquired translocation placing the coding sequence for M-CSF adjacent to the promoter of the collagen COL6A3 gene is found in these lesions. Though histologically benign, lesions can be locally aggressive and extend into adjacent bone and soft tissue. The other listed options are not part of the pathogenesis of this lesion.

A 55-year-old, previously healthy man has had episodes of pain and swelling of the right first metatarsophalangeal joint for the past year. These flare-ups usually occur after consumption of alcohol, typically port wine (Six Grapes). On physical examination, there is exquisite tenderness with swelling and erythema of the right first metatarsophalangeal joint. A joint aspiration is performed, and polarized light microscopy ( arrow in axis of red compensator) of the fluid obtained shows the finding in the figure, and many neutrophils in a small amount of fluid. Which of the following drugs is most likely to be useful as the first line of treatment? A Antibiotic B Corticosteroid C Nonsteroidal anti-inflammatory agent D Folate antagonist E Xanthine oxidase inhibitor

C Acute inflammation of the first MP joint, caused by precipitation of the needle-shaped negatively birefringent (yellow) uric acid crystals in the joint space, is typical of gout. Hyperuricemia is a sine qua non for the development of gout. Not all patients with hyperuricemia develop gout, and not all patients with gout have hyperuricemia, however. Other, ill-defined factors play a role in pathogenesis. Involvement of the big toe is classic, but other joints may be involved. Although attacks of gout are often precipitated by a heavy bout of alcohol consumption, liver damage (marked by elevated transaminases) is not a feature of gouty arthritis. Nonsteroidal anti-inflammatory drugs (NSAIDs) represent the first line of therapy for acute attacks. This is a chemically induced inflammatory reaction, not infectious, so antibiotics are not indicated. Glucocorticoids have a more pronounced effect upon chronic inflammatory conditions, and their continued use in joints will lead to degenerative arthritic changes. Methotrexate may be considered for treatment of rheumatoid arthritis. Xanthine oxidase inhibitors reduce uric acid formation and uricosuric agents help increase uric acid secretion, but they do not work acutely.

A 35-year-old man has had an outbreak of pruritic lesions over the extensor surfaces of the elbows and knees during the past month. He has a history of malabsorption that requires him to eat a special diet, but he has had no previous skin problems. On physical examination, the lesions are 0.4- to 0.7-cm vesicles. A 3-mm punch biopsy of one of the lesions over the elbow is performed and on microscopic examination shows accumulation of neutrophils at the tips of dermal papillae and formation of small blisters caused by separation at the dermoepidermal junction. Immunofluorescence studies show granular deposits of IgA localized to tips of dermal papillae. Laboratory studies show serum antigliadin antibodies. What is the most likely diagnosis? A Bullous pemphigoid B Contact dermatitis C Dermatitis herpetiformis D Discoid lupus erythematosus E Erythema multiforme F Pemphigus vulgaris

C Dermatitis herpetiformis can accompany celiac disease. The IgA or IgG antibodies formed against the gliadin protein in gluten that is ingested (commonly in wheat, rye, and barley grains) cross-react with reticulin. Reticulin is a component of the anchoring fibrils that attach the epidermal basement membrane to the superficial dermis. This explains the localization of the IgA to the tips of dermal papillae and the site of inflammation. A gluten-free diet may relieve the symptoms. Bullous pemphigoid can occur in older individuals, with antibody directed at keratinocytes to produce flaccid bullae, but there is no association with celiac disease. Contact dermatitis is most likely to be seen on the hands and forearms. It is a type IV hypersensitivity reaction without immunoglobulin deposition and would not persist for 1 month. Discoid lupus erythematosus is seen on sun-exposed areas and has the appearance of an erythematous rash. Erythema multiforme is a hypersensitivity response to infections and drugs; it produces macules and papules with a red or vesicular center, but it is probably mediated by cytotoxic lymphocytes and not by immunoglobulin deposition. Pemphigus vulgaris is an autoimmune disease in which IgG deposited in acantholytic areas forms vesicles that rupture to form erosions; it is not related to celiac disease.

A 30-year-old woman has experienced pain in the area of the left knee for 1 month. On physical examination, there is tenderness to palpation of the distal left thigh and knee. The area is firm, but there is no erythema or warmth. A radiograph of the left leg shows a 7-cm mass in the distal femoral epiphyseal area, with a "soap bubble" appearance. Microscopic examination of a curettage specimen of the lesion shows the findings in the figure. The lesion recurs in the next year; it is excised and does not recur again. What is the most likely diagnosis? A Chondrosarcoma B Enchondroma C Giant cell tumor D Osteitis fibrosa cystica E Osteoblastoma F Plasmacytoma

C Giant cell tumors typically arise in the epiphyses of long bones of individuals 20 to 40 years old; there is a slight female predominance. The tumors may recur after curettage. Although most are histologically and biologically benign, with multinucleated cells in a stroma predominantly composed of spindle-shaped mononuclear cells as shown in the figure, in rare cases, a sarcoma can arise in a giant cell tumor of bone. Chondrosarcomas are typically larger destructive lesions. Enchondromas are most often peripheral skeletal lesions involving the metaphyseal region of small tubular bones of the hands and feet. An osteoblastoma usually involves the spine. Osteitis fibrosa cystica is a complication of hyperparathyroidism. A plasmacytoma composed of neoplastic plasma cells is most often one lesion of multiple myeloma, more likely to occur in older adults.

A 27-year-old man develops acute pain and swelling of the left knee 5 days after an episode of urethritis. On physical examination, the left knee is swollen, warm, and tender to the touch. No other joints are affected. Laboratory examination of fluid aspirated from the left knee joint shows numerous neutrophils. A Gram stain of the fluid shows with gram-negative intracellular diplococci. No crystals are seen. Which of the following infectious agents is most likely responsible for his condition? A Borrelia burgdorferi B Haemophilus influenzae C Neisseria gonorrhoeae D Staphylococcus aureus E Treponema pallidum

C Gonorrhea should be considered the most likely cause of an acute suppurative arthritis in sexually active individuals; in some cases multiple joints can be involved. In men, a urethritis may occur with gonorrheal infection. Borrelia burgdorferi causes Lyme disease, characterized by chronic arthritis that may mimic rheumatoid arthritis. Haemophilus influenzae is a short, gram-negative rod that can cause osteomyelitis in children. Staphylococcus aureus is the most common cause of osteomyelitis, but the Gram stain would show gram-positive cocci. Treponema pallidum infection, also a sexually transmitted disease, can lead to syphilitic gummas in the tertiary phase of syphilis that may produce joint deformity. There is no preceding urethritis, however. Tertiary syphilis may be preceded years earlier by a primary syphilitic chancre.

A 33-year-old woman develops a skin rash on her face when she is outside in the sun for more than 1 hour. Laboratory studies show hemoglobin, 10.2 g/dL; hematocrit, 31.3%; platelet count, 126,800/mm 3 ; and WBC count, 3211/mm 3 . Urinalysis shows no glucose, but there is 3+ proteinuria and 2+ blood. Her ANA test result is positive with a titer of 1:2048 and a diffuse homogeneous immunofluorescent staining pattern. Which of the following complications is most likely to occur in this patient? A Bronchoconstriction B Cerebral lymphoma C Hemolytic anemia D Keratoconjunctivitis E Sacroiliitis F Sclerodactyly

C Patients with systemic lupus erythematosus (SLE) can develop anti-RBC antibodies, which can cause hemolytic anemia. Cytopenias, including leukopenia, thrombocytopenia, and anemia, also are common. Bronchoconstriction is a feature of bronchial asthma and can occur in allergies as a predominantly type I hypersensitivity reaction. Cerebral lymphomas are rare, but may occur in immunodeficient patients, particularly patients with AIDS. Keratoconjunctivitis can be seen in Sjögren syndrome as a result of decreased tear production from lacrimal gland inflammation. Sacroiliitis is a feature of many of the spondyloarthropathies, such as ankylosing spondylitis. Sclerodactyly is seen in scleroderma. When extensive, it is usually part of the spectrum of findings associated with diffuse scleroderma; when it involves only a few areas of the skin (e.g., just the hands), it is more likely to indicate limited scleroderma (CREST syndrome).

A 39-year-old woman has developed vesicular skin lesions over the past week. On physical examination, she has multiple, 0.2- to 1-cm vesicles and bullae on the skin of her scalp, axillae, groin, and knees. Many lesions appear to have ruptured, and a shallow erosion with a dried crust of serum remains. A biopsy specimen of an axillary lesion examined microscopically shows epidermal acantholysis and formation of an intraepidermal blister. The basal cell layer is intact. Which of the following additional tests is most likely to explain the pathogenesis of the patient's disease? A Darkfield microscopy of vesicular fluid B HLA genotyping C Immunostaining with antidesmoglein D Quantitation of serum IgE level E Viral culture of vesicular fluid

C Pemphigus vulgaris lesions are caused by IgG autoantibodies directed at an intercellular cement substance called desmoglein, giving a netlike appearance with immunofluorescence microscopy. The antibody deposition disrupts intercellular bridges, causing the epidermal cells to detach from each other (acantholysis). This action causes the formation of an intraepidermal blister. Staining with anti-IgG illuminates intercellular junctions at sites of incipient acantholysis. Darkfield microscopy is used almost exclusively to identify spirochetes in cases of syphilis. Some HLA types have an increased risk for some skin diseases, but this does not predict who will develop the disease. Type I hypersensitivity with IgE fixed to mast cells and urticaria does not produce an acantholytic vesicle. Herpes simplex viral infections can produce crops of vesicles, but such a wide distribution would be unusual.

A 47-year-old woman has been bothered for 20 years by recurring skin lesions over the elbows, knees, scalp, and lumbosacral area. These skin lesions are silvery to salmon-colored, 1- to 4-cm plaques with scaling. She has had increasing pain in her left hand and in her hips, more prominent on the left, over the past 2 years. On physical examination, she has yellow-brown discoloration with pitting of the fingernails. The distal interphalangeal joints of the left hand are slightly swollen and tender. There is minimal reduction in left hip mobility and no swelling or warmth to the touch. A radiograph of the left hip shows minimal joint space narrowing and surface erosion. Bone density is normal for age. During the next 10 years, the joint pain persists, but there is no joint destruction or deformity. She continues to have the same skin lesions. Which of the following is most likely to be seen on a biopsy specimen of her skin lesions? A Bandlike upper dermal infiltrate of lymphocytes B Epidermal spongiosis with dermal edema and eosinophils C Epidermal thinning with hyperkeratosis and parakeratosis D Focal keratinocyte apoptosis E IgG deposited along the dermal-epidermal junction

C Psoriasis with psoriatic arthritis has features of rheumatoid arthritis, but without significant joint destruction. Psoriasis is common, affecting 1% to 2% of individuals, and about 5% of these have psoriatic arthritis. For the remaining choices, there is no significant association with arthritis. A bandlike dermal infiltrate is typical of lichen planus, which produces pruritic violaceous plaques or papules, but tends to abate in 1 to 2 years. Epidermal spongiosis with eosinophilic infiltrates can be seen in acute eczematous dermatitis as part of a drug reaction. Focal keratinocyte apoptosis is seen in graft-versus-host disease. IgG deposition can be seen in systemic lupus erythematosus and in bullous pemphigoid.

A 53-year-old man noticed enlargement over 2 months in the skin lesion on the upper, outer area of his right arm, shown in the figure. Physical examination yields no other remarkable findings. Which of the following occupations is this man most likely to have had earlier in life? A Auto repair mechanic B Chemist in a factory C Lifeguard on the beach D Miner in a coal mine E Radiation oncologist at a cancer center

C The figure shows a malignant melanoma with abnormal ABCs: Asymmetry, irregular Borders, and variable Color of pigmentation. A quickly developing change in a pigmented lesion suggests the possibility of melanoma. Some melanomas are familial, arising from conditions such as dysplastic nevus syndrome. Most melanomas occur sporadically, however, and are related to sun exposure, as may occur in a lifeguard. The other listed occupations have no significant sun exposure.

A study of persons developing skin lesions following sun exposure is conducted. The lesions are not found on skin protected from ultraviolet light. Biopsies of involved skin show immunoglobulin G deposition along the dermal-epidermal junction, along with vacuolization of the basal layer and a perivascular lymphocytic infiltrate. No other organ involvement is present. Which of the following diseases do these patients most likely have? A Bullous pemphigoid B Celiac disease C Discoid lupus erythematosus D Dysplastic nevus syndrome E Toxic epidermal necrolysis

C The more benign discoid lupus involves just skin, unlike systemic lupus erythematosus, but is still a form of type III hypersensitivity with antigen-antibody complex deposition along the basement membrane of the epidermis. The other listed options are not associated with sun exposure. Bullous pemphigoid lesions occur at the dermal-epidermal junction from antibody deposition targeting type XVII collagen as a component of hemidesmosomes. Dermatitis herpetiformis associated with celiac disease has IgA antibodies deposited at tips of dermal papillae. Dysplastic nevi develop in relation to mutations in genes encoding for growth control proteins. Toxic epidermal necrolysis is a severe form of erythema multiforme mediated by cytotoxic CD8+ cells targeting epidermal basal cells. Early lesions of discoid lupus erythematosus appear as well-demarcated scaly purple macules or papules, and later expand into discoid plaques. Microscopically there is basal vacuolar degeneration, areas of epidermal atrophy, acanthosis, keratotic follicular plugging, basement membrane thickening, and superficial and deep perivascular lymphocytic infiltrates.

A 35-year-old man has noted a small bump on his upper trunk for the past 6 weeks. On physical examination, there is a solitary, 0.4-cm, flesh-colored nodule on the upper trunk. The dome-shaped lesion is umbilicated, and a curdlike material can be expressed from the center. This material is smeared on a slide, and Giemsa stain shows many pink, homogeneous, cytoplasmic inclusions. The lesion regresses over the next 2 months. Which of the following infectious agents most likely produced this lesion? A Histoplasma capsulatum B Human papillomavirus C Molluscum contagiosum D Staphylococcus aureus E Tinea corporis F Varicella-zoster virus

C The pink cytoplasmic inclusions, called molluscum bodies, are characteristic of this lesion. Immunocompromised individuals may have multiple, larger lesions. The infectious agent is a poxvirus. Disseminated fungal infections such as histoplasmosis involving skin are uncommon except in immunocompromised patients. Human papillomavirus (not a toad) is implicated in the appearance of verruca vulgaris, or the common wart. Staphylococcus aureus is implicated in the formation of the lesions of impetigo and pustular skin infections. There are a variety of superficial fungal infections caused by dermatophytes, such as Tinea corporis producing a scaling plaque. Varicella-zoster virus causes shingles, characterized by a dermatomal distribution of clear, painful vesicles.

A 78-year-old woman has had a constant, dull pain in her back that has persisted for more than a month. She is in no acute distress. On physical examination, there are no abnormal findings. Laboratory findings include creatinine, 0.9 mg/dL; urea nitrogen, 17 mg/dL; total protein, 6.8 g/dL; albumin, 4.2 g/dL; total bilirubin, 0.8 mg/dL; AST, 25 U/L; ALT, 29 U/L; calcium, 10.8 mg/dL; phosphorus, 2.3 mg/dL; and alkaline phosphatase, 228 U/L. What is the most likely cause of hypercalcemia? A Chondrosarcoma B Gouty arthritis C Metastatic carcinoma D Osteoarthritis E Osteoporosis F Paget disease of bone

C The prevalence of cancer increases with age. In the absence of other findings, a metastatic carcinoma involving bone should be suspected. In addition, parathyroid hormone-related peptide elaborated by neoplasms can be a cause for hypercalcemia of malignancy. Chondrosarcoma can occur over a wide age range, but these focal lesions involving bone are not likely to elevate serum calcium. Metastases to bone are far more common than primary bone malignancies. Biliary tract obstruction or infiltrative diseases of the liver are usually the cause for an elevated alkaline phosphatase of liver origin. Gouty arthritis may be accompanied by some local bone destruction, but without a marked increase in serum calcium and the lesions are typically in peripheral joints. Osteoarthritis produces pain, usually involving weight-bearing joints or hands, but has no abnormal laboratory markers. Although there is accelerated bone loss with osteoporosis, alkaline phosphatase and serum calcium are normal. Paget disease of bone is associated with an increased alkaline phosphatase, but not hypercalcemia.

A 32-year-old woman has noticed depigmented areas on her trunk that have waxed and waned for 3 months. She says that they do not itch or bleed, and are not painful. Physical examination shows variably sized, 0.3- to 1.2-cm macules over her upper trunk. The macules are lighter colored than the surrounding skin and have a fine, peripheral scale. Infection with which of the following organisms is most likely to produce these findings? A Epidermophyton species B Human papillomavirus C Malassezia furfur D Mycobacterium leprae E Propionibacterium acnes F Sarcoptes scabiei

C Tinea versicolor is a common condition caused by a superficial fungal infection of Malassezia furfur. The lesions can be lighter or darker than the surrounding skin. Epidermophyton, Trichophyton, and Microsporum genera are dermatophytic fungi best known as the cause of athlete's foot and jock itch. Human papillomavirus is best known as the cause of genital warts (condyloma acuminatum) and as a driving force behind cervical and anal squamous cell dysplasias. Mycobacterium leprae is the cause of Hansen disease, which can manifest with areas of skin anesthesia that predispose to repeated trauma. Infection with Propionibacterium acnes is a factor in the development of acne. Sarcoptes scabiei is the cause of scabies, which appears as pruritic reddish lesions.

A 43-year-old man has experienced malaise, fever, and a 4-kg weight loss over the past month. On physical examination, his blood pressure is 145/90 mm Hg, and he has mild diffuse abdominal pain, but no masses or hepatosplenomegaly. Laboratory studies include a serum urea nitrogen concentration of 58 mg/dL and a serum creatinine level of 6.7 mg/dL. Renal angiography shows right renal arterial thrombosis, and the left renal artery and branches show segmental luminal narrowing with focal aneurysmal dilation. During hemodialysis 1 week later, he experiences abdominal pain and diarrhea and is found to have melena. Which of the following serologic laboratory test findings is most likely to be positive in this patient? A ANA B C-ANCA C HIV D HBsAg E Scl-70 F RPR

D Classic polyarteritis nodosa (PAN) has segmental involvement of medium-sized arteries with aneurysmal dilation in the renal and mesenteric vascular beds (e.g., abdominal pain, melena). PAN can affect many organs at different times. Although the cause of PAN is unknown, about 30% of patients have hepatitis B surface antigen that presumably forms immune complexes that damage vascular walls. In contrast to microscopic polyangiitis, PAN has less of an association with anti-neutrophil cytoplasmic autoantibody (ANCA). A collagen vascular disease with a positive ANA test result, such as systemic lupus erythematosus, may produce a vasculitis, but not in the pattern seen here; the affected vessels are smaller. Vasculitis with HIV infection is uncommon. The Scl-70 autoantibody is indicative of scleroderma, which can produce renal failure. The rapid plasma reagin (RPR) is a serologic test for syphilis; an endaortitis of the vasa vasorum can occur in syphilis.

A 51-year-old woman has had bilateral diffuse pain in her thighs and shoulders for the past 6 weeks. She has difficulty rising from a chair and climbing steps. She has a faint violaceous rash around the orbits and on the skin of her knuckles. On physical examination, she is afebrile. Muscle strength is 4/5 in all extremities. Laboratory studies show serum creatine kinase of 753 U/L, and the ANA test result is positive with a titer of 1:160. Which of the following serologic tests is most specific for the diagnosis of her underlying condition? A Anti-cyclic citrullinated peptide B Anti-double-stranded DNA antibodies C Antihistone antibodies D Anti-Jo-1 antibodies E Anti-U1-ribonucleoprotein antibodies

D Dermatomyositis is a form of inflammatory myopathy in which capillaries are the primary target for antibody and complement-mediated injury. Anti-Jo-1 antibodies, although not present in most cases, are quite specific for inflammatory myopathies. The perivascular and perimysial inflammatory infiltrates result in peripheral muscle fascicular myocyte necrosis. The process is mediated by CD4+ cells and B cells. The violaceous, or heliotrope, rash is a characteristic feature of dermatomyositis. The antibody to cyclic citrullinated peptide is present in most patients with rheumatoid arthritis, which is accompanied by inflammatory destruction of joints, not muscle, although muscle may atrophy secondary to diminished movement. Anti-double-stranded DNA is specific for systemic lupus erythematosus (SLE), in which there can be myositis without significant inflammation or necrosis. Antihistone antibodies are associated with drug-induced SLE. The anti-U1-ribonucleoprotein antibodies suggest a diagnosis of mixed connective tissue disease, a condition that can overlap with polymyositis.

A 26-year-old woman has had bouts of joint pain for the past 2 years. She also has a rash on the cheeks and bridge of the nose. On physical examination, there is no joint swelling or deformity, although generalized lymphadenopathy is present. Laboratory studies indicate anemia, leukopenia, a polyclonal gammopathy, proteinuria, and hematuria. The serum ANA test result is positive at a titer of 1:1024 with a rim pattern identified by immunofluorescence. The light microscopic and immunofluorescent (with antibody to IgG) appearances of a skin biopsy specimen are shown in the figure. Which of the following is the best information to give this patient about her disease? A Blindness is likely to occur within 5 years B Avoid exposure to cold environments C Joint deformities will eventually occur D Chronic renal failure is likely to develop E Cardiac valve replacement will eventually be required

D Many patients with systemic lupus erythematosus (SLE) have glomerulonephritis, as evidenced by proteinuria with hematuria, and eventually develop renal failure. Blindness is uncommon in SLE. Raynaud phenomenon is associated with many autoimmune diseases, but it is most troublesome in scleroderma. Although synovial inflammation is common in SLE, joint deformity is rare. Libman-Sacks endocarditis associated with SLE tends to be nondeforming and limited, so there is minimal valve damage. It is now uncommon because of the use of corticosteroid therapy in the treatment of SLE.

A 22-year-old woman has been bothered by dryness in her mouth for the past 2 years. During this time, erythematous rashes have appeared on the skin of her face and upper neck. In the past 6 months, she has developed arthralgias. The fingers of her hands become pale and painful upon exposure to cold. On physical examination, she is afebrile. There are no joint deformities. Laboratory findings include a positive ANA test result, with a speckled pattern, and high titers of antibodies to U1-ribonucleoprotein (RNP). The serum creatinine is 1.1 mg/dL, and the urea nitrogen is 17 mg/dL. Which of the following diseases is most likely to produce these findings? A Dermatomyositis B Discoid lupus erythematosus C Limited scleroderma D Mixed connective tissue disease E Reactive arthritis F Sjögren syndrome G Systemic lupus erythematosus

D Mixed connective tissue disease (MCTD) can have some features of systemic lupus erythematosus (SLE), myositis, rheumatoid arthritis, scleroderma, and Sjögren syndrome. In contrast to SLE or diffuse scleroderma, serious renal disease is unlikely. Dermatomyositis causes muscle pain, and the rash is typically a subtle heliotrope rash with a violaceous appearance to the eyelids; Jo-1 antibody is a more typical finding. Discoid lupus erythematosus (DLE) is characterized by a rash similar to SLE, but with immune complex deposition only in sun-exposed areas of the skin, a positive ANA test result in a few cases, absence of anti-Smith or anti-double-stranded DNA antibodies, and absence of serious renal disease. Some cases of DLE can progress to SLE. In limited scleroderma (previously described as CREST syndrome), anticentromere antibody is often present. In reactive arthritis (with conjunctivitis, arthritis, and nongonococcal urethritis), there is often a positive serology for HLA-B27 and a history of either nongonoccocal urethritis with Chlamydia or gastrointestinal infection with Campylobacter, Salmonella, or Shigella organisms. In Sjögren syndrome, keratoconjunctivitis predominates, and antibodies to SS-A and SS-B are often present. The anti-Smith or anti-double-stranded DNA antibodies are more specific for SLE.

A 3-year-old child from Osaka, Japan, has developed a fever and a rash and swelling of her hands and feet over the past 2 days. On physical examination, her temperature is 37.8° C. There is a desquamative skin rash, oral erythema, erythema of the palms and soles, edema of the hands and feet, and cervical lymphadenopathy. The child improves after a course of intravenous immunoglobulin therapy. Which of the following is most likely to be a complication of this child's disease if it is untreated? A Asthma B Glomerulonephritis C Intracranial hemorrhage D Myocardial infarction E Pulmonary hypertension

D Mucocutaneous lymph node syndrome, or Kawasaki disease, involves large, medium-sized, and small arteries. Cardiovascular complications occur in 20% of cases and include thrombosis, ectasia, and aneurysm formation of coronary arteries. Asthma can be seen in association with Churg-Strauss vasculitis. Glomerulonephritis is a feature of anti-neutrophil cytoplasmic autoantibody (ANCA)-associated granulomatous vasculitis and of some autoimmune diseases such as systemic lupus erythematosus. Intracranial hemorrhage can occur with septic emboli to peripheral cerebral arteries, producing mycotic aneurysms that can rupture. Pulmonary hypertension can also complicate Takayasu arteritis, but is not as life-threatening as the coronary artery disease.

A 71-year-old man has experienced aching pain in the right knee, lower back, right distal fifth finger, and neck over the past 10 years. He notices that the joints feel slightly stiff in the morning, but this passes quickly. The pain is worse toward the end of the day. On physical examination, there is no joint swelling, warmth, or deformity. Some joint crepitus is audible on moving the knee. Laboratory studies show normal levels of serum calcium, phosphorus, alkaline phosphatase, and uric acid. What is the most likely cause of hypercalcemia? A Ankylosing spondylitis B Gouty arthritis C Multiple myeloma D Osteoarthritis E Pseudogout F Rheumatoid arthritis

D Osteoarthritis is a common problem of aging, and various joints, from large, weight-bearing joints to small joints, can be involved. Joint stiffness in the morning is a common feature, but it is minimal and quickly subsides. Ankylosing spondylitis causes back pain with deformity, but does not usually affect distal extremities. Gouty arthritis occurs in patients with elevated serum levels of uric acid, whereas pseudogout arises from calcium pyrophosphate dihydrate crystal deposition; both produce acute arthritis with pain and swelling. Multiple myeloma can produce lytic lesions in bone, but does not typically involve joints. Rheumatoid arthritis can involve large or small joints. It is typically associated with symmetric involvement of small joints of the hands and feet.

A 9-year-old boy has had pain in the area of the right hip for the past 3 weeks. On physical examination, his temperature is 38.2° C. There is swelling with marked tenderness to palpation in the area of the right hip, pain, and reduced range of motion. Radiographs of the pelvis and legs show areas of osteolysis and cortical erosion involving the femoral metaphysis, with adjacent soft-tissue swelling extending from the subperiosteal region, and apparent abscess formation. Which of the following organisms is most likely to produce these findings? A Haemophilus influenzae B Neisseria gonorrhoeae C Salmonella enterica D Staphylococcus aureus E Group B streptococcus

D Pyogenic osteomyelitis may arise from hematogenous dissemination of an infection. In children with no history of previous illnesses, Staphylococcus aureus is the most common causative organism. Haemophilus influenzae and group B streptococcal infections are most common in the neonatal period. Gonorrhea occasionally may disseminate and involve the bones (osteomyelitis) or joints (septic arthritis) of sexually active individuals. Salmonella infection involving bone is infrequent, except in patients with sickle cell anemia.

A 75-year-old man has noticed slowly enlarging lesions, similar to those shown in the figure, on his trunk over the past 20 years. One of the lesions is excised, and microscopic examination shows sheets of lightly pigmented basaloid cells that surround keratin-filled cysts. This lesion is sharply demarcated from the surrounding epidermis. What is the most likely diagnosis? A Basal cell carcinoma B Intradermal nevus C Melanoma D Seborrheic keratosis E Squamous cell carcinoma F Verruca vulgaris

D Seborrheic keratoses are flat, round, pigmented, sharply demarcated lesions and are benign tumors. They are composed of pigmented basaloid cells. Seborrheic keratoses are common in older individuals. The lesions gradually enlarge, but they are not painful and do not ulcerate. They mainly are a cosmetic problem. Basal cell carcinomas are nodular, slowly enlarging lesions most common on the head and trunk and are related to sun exposure. An intradermal nevus can produce a pigmented nodule, but microscopically it is composed of nests of small nevus cells, and the lesions increase minimally in size over time. Melanomas have very atypical spindle to epithelioid cells that invade the dermis; they tend to change in appearance over weeks to months, not years. The cells in a squamous cell carcinoma are atypical and may invade the dermis. A verruca vulgaris, or wart, also has a rough surface, but such lesions are more common on the hands and feet, and they may regress after several years.

A 58-year-old man with a diagnosis of chronic myeloid leukemia is treated with intensive chemotherapy. He goes into remission, but develops pain in the left wrist. On physical examination, there is swelling and warmth on palpation of the wrist. Polarized light microscopy of fluid aspirated from the wrist joint shows needle-shaped crystals that display negative birefringence. Which of the following processes most likely played an important role in the pathogenesis of the patient's wrist pain? A Abnormal platelet function with joint hemorrhages B Chemotherapeutic toxicity to the articular cartilage C Cytokine-induced synovial proliferation D Excessive production of uric acid E Leukemic cell infiltration of the synovium

D Secondary gout can occur in patients with leukemia, especially patients with a high leukocyte count (as occurs in chronic myeloid leukemia), who are treated with chemotherapeutic agents causing massive lysis of leukemic cell nuclei, and large amounts of urate are produced. The hyperuricemia leads to deposition of crystals in the joint space that triggers a local inflammatory response. Joint hemorrhages might be seen in patients with thrombocytopenia, but they more typically occur in patients with hemophilia, and neither induces needle-shaped crystal formation. Rhomboid cholesterol crystals are the most likely product of blood breakdown. Articular cartilage damage can be seen in any form of chronic arthritis, but is a prominent feature of osteoarthritis, not chemotherapy. Synovial proliferation also is a nonspecific change that is prominent in the inflammation of rheumatoid arthritis, not leukemia. Leukemic cells may infiltrate a variety of visceral organs, but usually not soft tissues.

A 44-year-old woman has developed skin lesions over her elbows and knees during the past year. The lesions start as 4-mm pustules with surrounding erythema but then evolve into 1- to 5-cm plaques that are covered with a silvery-white scale. The lesions appear first in areas of local trauma, but exposure to sunlight causes the lesions to regress. A biopsy of one lesion shows thinning of stratum granulosum with marked overlying parakeratotic scale containing microabscesses. Which of the following risk factors is most likely to be associated with her skin disease? A Atopy with elevated levels of IgE B Autoantibodies to epidermal desmoglein C Exposure of skin to plant allergens D Inheritance of certain HLA-Cw alleles E Herpes simplex virus infection

D She has psoriasis, and the appearance of lesions with trauma is known as the Koebner phenomenon. Though two thirds of affected persons have the HLA-Cw∗0602 allele, only 10% of all persons with this allele develop psoriasis. There is abnormal CD4+ and CD8+ lymphocyte activation with release of many cytokines, including tumor necrosis factor, which mediate the skin damage. Atopy increases the risk for allergic reactions mediated by type I hypersensitivity reactions, often with urticarial skin lesions that dissipate within hours. Autoantibodies to desmoglein in pemphigus vulgaris cause suprabasal blister formation. Plant allergens such as urushiol in poison oak cause a contact dermatitis that typically fades in days. Herpes simplex virus infection can produce vesicular eruptions, or it may underlie erythema multiforme manifested by targetoid lesions.

A 47-year-old woman has had an ocular burning sensation with increasing blurring of vision for the past 5 years. On physical examination, she has keratoconjunctivitis. Atrophy of the oral mucosa, with buccal mucosal ulcerations, also is present. A biopsy specimen of the lip shows marked lymphocytic and plasma cell infiltrates in minor salivary glands. Antibodies to which of the following are most likely to be identified on laboratory testing? A Centromere B Double-stranded DNA C DNA topoisomerase (Scl-70) D SS-B E U1-RNP

D Sjögren syndrome primarily involves salivary and lacrimal glands. Antibodies to SS-B are found in the majority of these patients. Anti-double-stranded DNA is a specific autoantibody for systemic lupus erythematosus. Anticentromere antibody is seen most often in limited systemic sclerosis, whereas anti-DNA topoisomerase most often appears with diffuse systemic sclerosis. U1-RNP is a marker for mixed connective tissue disease (MCTD).

An 8-year-old boy complains of left leg pain for 3 days. On physical examination, his temperature is 38.9° C, and he exhibits irritability when his left leg is moved. A radiograph of the left leg shows changes suggesting acute osteomyelitis in the proximal portion of the left femur. Culture of the infected bone is most likely to grow which of the following organisms? A Hemophilus influenzae B Neisseria gonorrhoeae C Salmonella enterica D Staphylococcus aureus E Streptococcus pneumoniae

D Staphylococcus aureus is the most common cause of osteomyelitis in children and adults. Infections with H. influenzae may occur in children less than 2 years of age, but vaccination has reduced the incidence of such cases. Both organisms can cause congenital infections. Gonorrhea as a cause of acute osteomyelitis should be considered in sexually active adults, and may be passed to the fetus in the birth canal, but typically involve the eyes. Salmonella osteomyelitis is most characteristic of individuals with sickle cell anemia. Pneumococcal osteomyelitis is uncommon.

A 39-year-old woman goes to her dentist for a routine checkup. The dentist notes that she has 0.2- to 1.5-cm scattered, white, reticulated areas on the buccal mucosa. The woman says that these lesions have been present for 1 year. She also has multiple 0.3-cm purple, pruritic papules on each elbow. A biopsy specimen of a skin lesion is taken and microscopically shows a bandlike infiltrate of lymphocytes at the dermal-epidermal junction with degeneration of basal keratinocytes. Which of the following is the most likely outcome for this woman? A Disseminated opportunistic infection B Multiple allergies to medications C Progression to chronic renal disease D Resolution of the lesions E Skin adnexal tumor development

D The classic "pruritic, purple, polygonal papules" of lichen planus are present, with the distinctive bandlike infiltrate of lymphocytes at the dermal-epidermal junction. Cytotoxic CD8+ cells are reacting to antigens in the basal layer and dermoepidermal junction. The lesions of lichen planus are typically self-limited, although the course can run for several years. Oral lesions may persist longer. There is risk for squamous cell carcinoma in chronic lesions. Although a lymphocytic infiltrate is present, an infection or autoimmunity is not implicated. A drug eruption would not last this long, and lichen planus is not a hypersensitivity reaction. Lesions of erythema multiforme are more likely to follow infections, drugs, autoimmune diseases, and malignancies. Skin adnexal tumors are typically benign; often have apocrine differentiation; and arise in areas were apocrine glands are prevalent, such as scalp and axilla. They are generally not associated with other diseases.

A 31-year-old woman has had increasing peripheral edema, pleuritic chest pain, and an erythematous rash for the past 6 months. Laboratory studies show increasing serum creatinine, and urinalysis shows proteinuria with RBC casts. A renal biopsy is performed, and the light microscopic appearance of the PAS-stained specimen is shown in the figure, compared with normal. Which of the following autoantibodies when present is specific for this patient's condition? A Centromere B Cyclic citrullinated polypeptide C DNA topoisomerase I D Smith E U1-RNP

D The figure shows the so-called wire loop glomerular capillary lesions of lupus nephritis. Anti-Smith and anti-double-stranded DNA are more specific for systemic lupus erythematosus, but sensitivity is low: anti-Smith is present in only 25% of SLE cases. Anticentromere antibody is seen most often with limited scleroderma, whereas anti-DNA topoisomerase I is found with diffuse scleroderma. Cyclic citrullinated polypeptide and rheumatoid factor are found most often with rheumatoid arthritis. Anti-U1-RNP can be found with mixed connective tissue disease.

A 27-year-old woman has developed areas of scaling skin over the past month. On physical examination there are 1- to 3-cm light pink plaques covered with silvery scale on her arms and torso. A punch biopsy of one lesion examined microscopically shows keratinocyte nuclei retained within cells in the stratum corneum. Which of the following descriptive terms best applies to this microscopic finding? A Acanthosis B Dyskeratosis C Hyperkeratosis D Parakeratosis E Spongiosis

D The lesions are those of psoriasis, driven by T lymphocyte elaboration of cytokines driving cellular proliferation so that the normal sequence of keratinocyte maturation is disrupted. Acanthosis refers to epidermal hyperplasia, in which the thickness of the entire epidermis is increased. Dyskeratosis refers to premature keratinization below the stratum granulosum. Hyperkeratosis is increased thickness of stratum corneum with abnormal keratin; a callus would refer to thickened stratum corneum in response to mechanical forces, such as the hands of a carpenter working with tools, or the soles of the feet of a child going barefoot. Spongiosis is edema between the cells of the epidermis.

A 70-year-old man complains of right hip and thigh pain of 8 months' duration. On physical examination, he has reduced range of motion in both hips, but there is no tenderness or swelling on palpation. Radiographs of the pelvis and right leg show sclerotic, thickened cortical bone with a narrowed joint space near the acetabulum. Laboratory studies show a serum alkaline phosphatase level of 173 U/L, calcium of 9.5 mg/dL, and phosphorus of 3.4 mg/dL. The figure shows the representative microscopic appearance of his pelvic bone. Which of the following conditions is most likely to produce these findings? A Degenerative osteoarthritis B Hyperparathyroidism C Osteochondroma D Paget disease of bone E Vitamin D deficiency

D The mosaic pattern of lamellar bone in the figure is characteristic of osteitis deformans (Paget disease of bone). This disease has three phases. Early in the course, there is a lytic phase, followed by the more classic mixed phase of osteosclerosis and osteolysis, leading to the appearance of a "mosaic" of irregular bone. A sclerotic "burnt-out" phase then ensues. Elderly white people are most often affected, and the disease progresses over many years. Joints adjacent to affected bone manifest osteoarthritis with chronic pain from joint erosion. Osteitis fibrosa cystica is seen in hyperparathyroidism. An osteochondroma is a tumorlike projection of bone capped by cartilage that protrudes from the metaphyseal region of a long bone. Osteomalacia results in osteopenia from vitamin D deficiency in an adult.

A 10-year-old girl has multiple excoriations that have appeared on the skin of her hands over the past week. The child reports that she scratches her hands because they itch. Physical examination shows multiple 0.2- to 0.6-cm linear streaks in the interdigital regions. Treatment with a topical lindane lotion resolves the condition. Which of the following organisms is most likely responsible for these findings? A Ixodes scapularis B Poxvirus C Staphylococcus aureus D Sarcoptes scabiei E Tinea corporis

D The small scabies mites burrow through the stratum corneum to produce the linear lesions, and the mites along with their eggs and feces produce intense pruritus. Scabies is easily transmitted by contact and typically occurs in community outbreaks. Ixodes scapularis is the tick that is the vector for Borrelia burgdorferi organisms, which cause Lyme disease and erythema chronicum migrans. Molluscum contagiosum is caused by a poxvirus that produces a localized, firm nodule. The erythematous macules and pustules of impetigo in children are often caused by staphylococcal and group A streptococcal infection. Tinea corporis is a superficial dermatophytic fungal infection that can produce erythema and crusting.

A 30-year-old man has had cramping abdominal pain and bloody diarrhea for the past 4 days. On physical examination, there is diffuse tenderness on palpation of the abdomen. Bowel sounds are present. There are no masses and no organomegaly. A stool culture is positive for Shigella flexneri. The episode resolves spontaneously within 1 week after onset. Six weeks later, he has increasingly severe lower back pain. Physical examination now shows stiffness of the lumbar spine and tenderness of the sacroiliac joints. He is treated with nonsteroidal anti-inflammatory agents. Two months later, the back pain recurs, and he complains of redness of the right eye and blurred vision. Serologic testing for which of the following is most likely to be positive in this patient? A Borrelia burgdorferi B Chlamydia trachomatis C Epstein-Barr virus D HLA-B27 E Rheumatoid factor

D This patient developed enteritis-associated arthritis affecting the lumbar and sacroiliac joints several weeks after Shigella dysentery. He subsequently developed conjunctivitis and, most likely, uveitis. This symptom complex is a classic representation of a cluster of related disorders called seronegative spondyloarthropathies. This cluster includes ankylosing spondylitis, reactive arthritis, psoriatic arthritis, and enteropathic arthritis (as in this case). A common feature is a very strong association with the HLA-B27 genotype. Despite some similarities with rheumatoid arthritis, these patients invariably have a negative test result for rheumatoid factor. Urethritis caused by Chlamydia trachomatis can trigger reactive arthritis, another form of seronegative spondyloarthropathy. Such infection precedes the onset of arthritis, however. There is no relationship between infection with Borrelia burgdorferi, the causative agent of Lyme disease, and reactive arthritis in individuals testing positive for HLA-B27. Similarly, Epstein-Barr virus infection is not a trigger for these disorders.

A 15-year-old boy has been hospitalized multiple times since childhood as a result of painful abdominal crises. He has had pain in his right hip region for the past week. On physical examination, there is marked tenderness and swelling to palpation over the right hip. Laboratory studies show hemoglobin of 8.5 g/dL, hematocrit of 25.7%, platelet count of 199,900/mm 3 , and WBC count of 12,190/mm 3 . Examination of the peripheral blood smear shows sickled erythrocytes, and nucleated RBCs. A radiograph of the pelvis and right upper leg shows acute inflammatory changes in the femoral head and metaphysis of the right proximal femur. Which infectious agent is most likely responsible for these findings? A Group B streptococcus B Klebsiella pneumoniae C Mycobacterium tuberculosis D Salmonella enterica E Staphylococcus aureus

D Though Staphylococcus aureus infection is responsible for 80% to 90% of all cases of osteomyelitis in which an organism can be cultured. Salmonella osteomyelitis is especially common, however, in patients with sickle cell anemia. Group B streptococcal infections causing osteomyelitis are most common in neonates. Klebsiella pneumoniae osteomyelitis may rarely be seen in adults with urinary tract infections caused by this organism. Tuberculosis is a rare cause of osteomyelitis in adults who have had active pulmonary disease with dissemination, most likely because of a poor immune response.

A 69-year-old woman has been bothered by a discolored area of skin on her forehead that has not faded during the past 3 years. On physical examination, there is a 0.8-cm red, rough-surfaced lesion on the right forehead above the eyebrow. A biopsy specimen examined microscopically shows basal cell hyperplasia. Some of the basal cells show nuclear atypia associated with marked hyperkeratosis and parakeratosis with thinning of the epidermis. The upper dermal collagen and elastic fibers show homogenization with elastosis. What is the most appropriate treatment option for this patient? A Apply hydrocortisone cream B Reduce intake of dietary fat C Surgically excise the lesion with wide margins D Take antioxidants E Wear a hat outdoors

E Actinic keratoses are premalignant lesions associated with sun exposure. Older individuals are more likely to have actinic keratoses because of greater cumulative sun exposure, not because of aging alone. More extensive actinic keratoses may be treated with 5-fluorouracil applied topically. Hydrocortisone can alleviate the symptoms of many dermatologic conditions, but it cannot reverse actinic damage. Decreasing dietary fat is always a good idea, but it does not have much effect on the skin of the face. A surgical excision with wide margins is indicated for a more aggressive lesion, such as malignant melanoma. Many drugs can cause acute eczematous dermatitis and erythema multiforme, but not skin cancer. Antioxidants are touted for slowing the effects of aging, but they cannot reverse actinic damage.

A 35-year-old Sami man from Finland has increasing lower back pain for 6 years. On examination he has decreased mobility of his spine. Radiographs show loss of lumbar lordosis, narrowing with sclerosis of sacroiliac joints bilaterally, and syndesmophyte formation at the junction of vertebrae and annulus fibrosus of intervertebral discs. Which of the following complications is most likely to develop in this man? A Glomerulonephritis B Hepatitis C Malabsorption D Meningitis E Uveitis

E Ankylosing spondyloarthritis has a very strong association with the HLA-B27 genotype that has a high prevalence among the native peoples of the circumpolar arctic and subarctic regions of Eurasia and North America. This progressive disease typically involves the lower back and pelvis. The radiographic feature of bamboo spine is characteristic. Spondyloarthropathies can have extra-articular manifestations such as anterior uveitis and aortitis. Glomerulonephritis is most likely a complication of collagen vascular diseases such as systemic lupus erythematosus. Autoimmune hepatitis is not related to joint diseases. Malabsorption is more likely to accompany diarrhea with enteropathic arthritis with infectious etiology. Meningitis may accompany Lyme disease.

A 6-year-old boy developed 1- to 2-cm erythematous macules and 0.5- to 1-cm pustules on his face over the past week. During the next 2 days, some of the pustules break, forming shallow erosions covered by a honey-colored crust. New lesions form around the crust. The boy's 40-year-old uncle develops similar lesions after visiting for 1 week during the child's illness. Removal of the crusts from the boy's face is followed by healing within 1 week. The uncle does not seek medical care, and additional pustules form at the periphery of the crusts. Which of the following conditions most likely explains these findings? A Acne vulgaris B Bullous pemphigoid C Contact dermatitis D Erythema multiforme E Impetigo F Lichen planus

E Impetigo is a superficial infection of skin that produces shallow erosions. These erosions are covered with exuded serum that dries to give the characteristic honey-colored crust. Cultures of the lesions of impetigo usually grow coagulase-positive Staphylococcus aureus or group A β-hemolytic streptococcus. The lesions are highly infectious. Acne vulgaris is typically seen during adolescence and produces pimples and pustules, but not crusts. Bullous pemphigoid can occur in older individuals with antibody directed at keratinocytes to produce flaccid bullae. Contact dermatitis is most likely to be seen on the hands and forearms. Erythema multiforme is a hypersensitivity response to infections and drugs that produces macules and papules with a red or vesicular center. Lichen planus appears as violaceous papules and plaques.

An 82-year-old man has had progressively worsening lower back, bilateral hip, and right shoulder pain for the past 6 years. He reports that he has had to buy larger hats. On physical examination, there is no joint swelling, erythema, warmth, or tenderness, but the range of motion is reduced. Radiographs show narrowing of joint spaces with adjacent bony sclerosis. A skull radiograph shows thickening but diminished density of the skull bones. A bone biopsy specimen at the iliac crest shows a loss of normal trabeculae, with a mosaic pattern and increased numbers of osteoclasts and osteoblasts. Which of the following complications is the patient most likely to experience as a result of this condition? A Ankylosing spondylitis B Enchondromatosis C Fibrous dysplasia D Osteoid osteoma E Osteosarcoma

E In 5% to 10% of patients with severe polyostotic Paget disease, sarcomas (including osteosarcoma) can arise years later in bone affected by the disease. Mutations in SQSTM1 are found in both familial and sporadic cases of Paget disease of bone. Ankylosing spondylitis involves the spine and carries no risk of malignancy. An enchondroma does not arise in the setting of Paget disease. Fibrous dysplasia is a focal developmental defect of bone seen at a younger age. An osteoid osteoma is a small benign cortical bone lesion that can produce severe pain in children and young adults.

A 34-year-old woman has experienced malaise, fatigue, and joint pain for the past 5 months. She has had progressive loss of joint motion, making it more difficult to walk and to use her hands. On physical examination, the joint involvement is symmetric, and most of the affected joints are in the hands and feet. The involved joints are swollen and warm to the touch. The second and third digits on the right hand have a swan neck deformity, and there is ulnar deviation of both hands. Reconstructive surgery is performed on her right hand. Microscopic views of the excised joint capsule tissue are shown in the figure. Which of the following serologic laboratory test findings is most likely to be positive in this patient? A Anti-DNA topoisomerase I antibody B Antinuclear antibody C Borrelia burgdorferi D Campylobacter jejuni E Citrullinated peptides

E In addition to rheumatoid factor, serology for cyclic citrullinated peptides (CCPs) has specificity for rheumatoid arthritis and may indicate chronicity of the disease. The immunologically mediated damage leads to chronic inflammation with synovial proliferation, shown in the figure, with pannus formation that gradually erodes and destroys the joints, resulting in joint deformity. Typically, the joint involvement is bilateral and symmetric, and small joints are often involved. Anti-topoisomerase I has specificity for scleroderma (systemic sclerosis). Antinuclear antibodies can be found in a variety of autoimmune diseases, including rheumatoid arthritis, but lack specificity. Lyme disease, caused by Borrelia burgdorferi infection, can produce a chronic arthritis that can destroy cartilage, but larger joints are usually involved. Campylobacter spp. may be related to cases of reactive arthritis.

A 33-year-old man has noticed a lump over his right flank. The lump is painless and has enlarged slowly over the past 3 years. On physical examination, a soft 2-cm nodule is palpable in the subcutis of the right flank above the iliac crest. The lesion is excised. Grossly, it is circumscribed and has a uniformly yellow cut surface. Which of the following is the most likely prognosis for this lesion? A Antibiotic therapy will be needed B Family members will develop similar lesions C Metastases to regional lymph nodes will occur D More skin lesions will occur over time E No recurrence is expected

E Lipoma is the most common benign soft-tissue neoplasm. Such masses are extremely well differentiated and discrete. Multiple lipomas may be seen in some familial cases, but these are rare. These benign tumors do not metastasize; mesenchymal neoplasms do not often metastasize through lymphatics. Recurrence of some atypical lipomas or liposarcomas is possible, but benign lipomas do not recur. Secondary infection of this uncomplicated excision procedure is unlikely.

A 38-year-old man has had chronic leg pain for the past 4 months. He passed a urinary tract calculus a month ago. On physical examination, there is local swelling with tenderness just below the right patella. A radiograph of the right lower leg shows a 4-cm cystic area in the right tibial diaphysis without erosion of the cortex or soft-tissue mass. Laboratory studies show serum calcium is 12.6 mg/dL, and phosphorus is 2.1 mg/dL. A biopsy specimen of the lesion is taken and microscopically shows increased osteoclasts and fibroblast proliferation. Which of the following underlying conditions is most likely to account for these findings? A Chronic glomerulonephritis B Chronic osteomyelitis C Giant cell tumor of bone D Paget disease of bone E Parathyroid adenoma

E Parathyroid adenomas secrete parathyroid hormone (PTH) and cause primary hyperparathyroidism with high serum calcium and low phosphate. Excessive secretion of PTH activates osteoclastic resorption of bone. Microfractures within the areas of bone resorption give rise to hemorrhages; this causes an influx of macrophages and, ultimately, reactive fibrosis. These lesions can become cystic, and they are sometimes called brown tumor of bone. Because they contain osteoclasts and fibroblasts, these lesions can be confused with primary bone neoplasms, such as giant cell tumor of bone. Chronic osteomyelitis rarely produces such a discrete lesion. Secondary hyperparathyroidism is seen in patients with chronic renal failure, but the serum phosphorus should be high. Giant cell tumors occur in the epiphysis or metaphysis, however, and they contain plump stromal cells, not fibroblasts. This patient is too young to have Paget disease of bone, which may be osteolytic in its early phase, but is not associated with hypercalcemia.

A 28-year-old man has had waxing and waning of the lesions shown in the figure for 10 years. The scalp, lumbosacral region, and glans penis also are affected. For the past 2 years, he has had chronic arthritis in the hips and knees. Which of the following physical findings would most likely be present in this patient? A Friction rub B Guaiac-positive stool C Hyperreflexia D Hypertension E Nail changes

E Psoriasis is a chronic skin condition with marked epithelial hyperplasia and parakeratotic scaling. CD4+ lymphocytes, as part of a T H 17 and T H 1 response, elaborate cytokines that promote cell proliferation, and CD8+ cells cause cellular damage. Nail changes, such as yellow-brown discoloration, pitting, dimpling, and separation of the nail plate from the nail bed (onycholysis), affect about one third of patients. Other manifestations of psoriasis include arthritis (resembling rheumatoid arthritis), myopathy, enteropathy, and atherosclerotic heart disease. A friction rub from fibrinous pericarditis does not occur in psoriasis because mesothelial surfaces are not involved. Gastrointestinal mucosal involvement with hemorrhage is not a feature of psoriasis. Joint laxity with hyperreflexia is a feature of Ehlers-Danlos syndrome. Renal disease and hypertension are not typically the result of psoriasis.

A 6-year-old boy complains of discomfort in the right upper neck that has worsened over the past 6 months. On physical examination, a 5-cm firm mass is palpable in the right lateral neck. The mass is not tender or warm. A biopsy is taken and the microscopic appearance of this mass is shown in the figure. Immunohistochemical staining for which of the following antigens is most likely to be positive in the cells of this lesion? A CD3 B Cytokeratin C Dystrophin D Neuron-specific enolase E Vimentin

E Rhabdomyosarcoma is the most common sarcoma in children. Sarcomas mark with antibody to vimentin, an intermediate cytoplasmic filament, with immunohistochemistry. Note the neoplastic spindle-shaped but markedly pleomorphic cells that have pink cytoplasm with a hint of striations mimicking skeletal muscle cells. CD3 is a T lymphocyte marker. Cytokeratin is a marker for tumors of epithelial origin (e.g., carcinomas). Dystrophin is a membrane-stabilizing protein in striated muscle; it is absent in Duchenne muscular dystrophy. Neuron-specific enolase is a marker of neoplasms with neural differentiation.

A 53-year-old man with idiopathic dilated cardiomyopathy underwent orthotopic heart transplantation. During the next 5 years, he had two episodes of minimal cellular rejection, which were adequately treated by an increase in immunosuppressive therapy. He has developed multiple skin lesions on the face and upper trunk over the past 6 months. On physical examination, the lesions are similar to the lesion shown in the figure. Some of the larger lesions have ulcerated. A biopsy specimen is most likely to identify which of the following lesions? A Dermatofibroma B Erythema multiforme C Lichen planus D Psoriasis E Squamous cell carcinoma

E Risk factors for squamous cell carcinoma include ultraviolet light exposure, scarring from burn injury, irradiation, and immunosuppression. This patient was immunosuppressed to prevent graft rejection, and the immune dysregulation favors carcinogenesis in transplant patients. Squamous cell carcinomas also arise in rare disorders of DNA repair, such as xeroderma pigmentosum. A dermatofibroma is typically solitary, firm, and dermally located. Erythema multiforme is a hypersensitivity response to infections or drugs; the lesions have multiple forms, including papules, macules, vesicles, and bullae. Lichen planus is a self-limited inflammatory disorder that manifests as "purple, pruritic, polygonal papules," not as elevated ulcerated lesions. Psoriasis is an inflammatory dermatosis that can be associated with underlying arthritis, myopathy, enteropathy, or atherosclerotic heart disease.

A 19-year-old woman has a skin rash involving her face and scalp for the past 9 months. On examination there are 0.5- to 1.5-cm plaques with erythema and edema. A punch biopsy is taken and on microscopic examination shows follicular plugging. There is positive immunofluorescence at the dermal-epidermal junction with staining for IgG and complement C3. Which of the following pathogenic mechanisms result from TLR engagement by self nucleic acids? A Mast cell degranulation B Molecular mimicry C Release of sequestered antigens D T H 17 elaboration E Type I interferon elaboration

E She has discoid lupus erythematosus, with skin lesions similar to those of systemic lupus erythematosus (SLE), but with systemic involvement much less likely to occur. Self nucleic acids mimic their microbial counterparts, and in conjunction with TLRs, they incite type I interferon production to activate dendritic cells and B cells. They also promote a T H 1 response, which contributes to autoimmunity. Mast cell degranulation with type I hypersensitivity may produce an urticarial rash, but without autoimmune antigen-antibody complex deposition. Molecular mimicry may follow an infection and lead to polyclonal B cell activation, but autoimmune diseases involve self antigens, not exogenous antigens. Release of sequestered antigens is the mechanism for sympathetic ophthalmia. A T H 17 response may be present in chronic inflammatory conditions such as inflammatory bowel diseases with macrophage and neutrophilic infiltrates.

A 66-year-old woman has experienced pain in the area around the left knee for the past 6 weeks. She can recall no trauma to the leg. On physical examination, no mass is palpable; there is no warmth or swelling, and there is no loss of range of motion. MRI shows a well-circumscribed 4-cm mass superior and inferior to the patella. The mass is within soft tissue, without bony erosion. A biopsy of the mass is obtained and on microscopic examination shows a biphasic pattern of spindle cells and epithelial cells forming glands. Karyotypic analysis of tumor cells shows a t(X;18) translocation. What is the most likely diagnosis? A Desmoid tumor B Leiomyosarcoma C Mesothelioma D Osteoblastoma E Synovial sarcoma

E Synovial sarcomas account for 10% of all adult sarcomas and can be found around a joint or in deep soft tissues because they arise from mesenchymal cells, not synovium. Most synovial sarcomas show the t(X;18) translocation. A desmoid tumor is a fibromatosis composed of fibroblasts and collagen. Leiomyosarcomas do not have a biphasic pattern microscopically and are rarely seen in soft tissues. A mesothelioma can be biphasic, but it more typically arises in the pleura, or less commonly the mesothelial surface of peritoneum or pericardium. An osteoblastoma is a bone mass that arises in the epiphyseal region.

A 32-year-old woman has had coldness and numbness in her arms and decreased vision in the right eye for the past 5 months. On physical examination, she is afebrile. Her blood pressure is 100/70 mm Hg. Radial pulses are not palpable, but femoral pulses are strong. She has decreased sensation and cyanosis in her arms, but no warmth or swelling. A chest radiograph shows a prominent border on the right side of the heart and prominence of the pulmonary arteries. Laboratory studies show serum glucose, 74 mg/dL; creatinine, 1 mg/dL; total serum cholesterol, 165 mg/dL; and negative ANA test result. Her condition remains stable for the next year. Which of the following is the most likely diagnosis? A Aortic dissection B Kawasaki disease C Microscopic polyangiitis D Syphilis E Takayasu arteritis F Thromboangiitis obliterans

E Takayasu arteritis leads to "pulseless disease," because of involvement of the aorta (particularly the arch) and branches such as coronary, carotid, and renal arteries, which results in granulomatous inflammation, aneurysm formation, and dissection. Fibrosis is a late finding, and the pulmonary arteries also can be involved. Aortic dissection is an acute problem that, in older adults, is driven by atherosclerosis and hypertension, although this patient is within the age range for complications of Marfan syndrome, which causes cystic medial degeneration of the aorta. Kawasaki disease affects children and is characterized by an acute febrile illness, coronary arteritis with aneurysm formation and thrombosis, skin rash, and lymphadenopathy. Microscopic polyangiitis affects arterioles, capillaries, and venules with a leukocytoclastic vasculitis that appears at a similar stage in multiple organ sites (in contrast to classic polyarteritis nodosa, which causes varying stages of acute, chronic, and fibrosing lesions in small to medium-sized arteries). Tertiary syphilis produces an endaortitis with proximal aortic dilation. Thromboangiitis obliterans (Buerger disease) affects small to medium-sized arteries of the extremities and is strongly associated with smoking.

A 60-year-old woman noted the change seen in the figure (Raynaud's) while she was driving to work one morning. There was associated pain and numbness. Within 20 minutes after entering the warm office building, these problems disappeared. What pathologic process has most likely led to these findings? A Calcification B Hypertension C Thrombosis D Vasculitis E Vasoconstriction

E The "red, white, and blue" changes shown represent Raynaud phenomenon, which can be a primary exaggerated vasoconstriction without an underlying disease. In older persons, an underlying disease, such as an autoimmune disease, should be sought. Hyperviscosity may underlie this phenomenon. In younger persons it is "primary" and likely vasomotor hyperreactivity. Calcification with medial calcific sclerosis tends to involve arteries that are small and muscular, but larger than those of hands or feet; it is often an incidental finding on a radiograph. Hypertension may drive atherosclerosis, but not marked vasoconstriction. Thrombosis is unlikely to develop and subside so quickly. Vasculitis likewise is not an evanescent phenomenon.

A 23-year-old man has had pain in the area of the right knee for the past year. On physical examination, there is point tenderness in a 2-cm focal area just below the patella laterally over the tibia. A radiograph of the right leg shows a 3-cm, broad-based excrescence projecting from the metaphyseal region of the upper tibia. The lesion is excised. The figure shows the gross appearance of the sectioned lesion. What is the most likely diagnosis? A Enchondroma B Fibrous dysplasia C Giant cell tumor D Osteoblastoma E Osteochondroma

E The figure shows an osteochondroma with glistening cartilaginous cap overlying cancellous bone. This tumorlike condition, also called an exostosis, is benign and, when solitary, is essentially an incidental finding because a sarcoma rarely arises from an osteochondroma. Multiple osteochondromas can be part of an inherited syndrome, however, and onset can be in childhood, accompanied by bone deformity and an increased risk of development of a sarcoma. Both hereditary and sporadic osteochondromas have loss of heterozygosity in EXT1 or EXT2 genes. An osteochondroma is a projection of the cartilaginous growth plate with proliferation of mature bone capped by cartilage. When skeletal growth ceases, osteochondromas tend to cease proliferation as well. They may produce local irritation and pain. An enchondroma is a benign tumor of hyaline cartilage that arises in the medullary space of young adults. Fibrous dysplasia is a localized area of developmental arrest of bone formation. A giant cell tumor is a benign but locally aggressive lesion that arises in the epiphysis of the long bones of young adults and has a "soap bubble" radiographic appearance. An osteoblastoma is a large osteoid osteoma, which can arise in epiphyseal lesions and cause intense pain.

A 16-year-old boy has had pain around the right knee for the past 3 months. There are no physical findings except for local pain over the area of the distal right femur. A radiograph of the right leg shows an ill-defined mass involving the metaphyseal area of the distal right femur, and there is elevation of the adjacent periosteum. A bone biopsy specimen is obtained and on microscopic examination shows large, hyperchromatic, pleomorphic spindle cells forming an osteoid matrix. Which of the following tumor suppressor genes is most likely to be mutated in this boy? A APC B BRCA1 C NF1 D PTEN E RB

E The osteoid production by a sarcoma is diagnostic of osteosarcoma. The metaphyseal location in a long bone, particularly in the region of the knee, is consistent with osteosarcoma, as is the presentation in a young individual. Sporadic cases of osteosarcoma must have loss of both alleles of RB1 , whereas in familial cases there is inheritance of one bad copy, and in those cases retinoblastomas are likely to appear first. Mutations of TP53 are found in many cancers, including osteosarcomas, and with the rare familial Li-Fraumeni syndrome. APC mutations are seen in association with colonic adenocarcinomas. BRCA1 mutations are associated with breast and ovarian cancers. NF1 mutations lead to neurofibromatosis type 1, with many neural and soft tissue tumor types, but less likely primary bone tumors. PTEN mutations are most often associated with endometrial and prostate cancers.

A 29-year-old woman has had fever and arthralgias for the past 2 weeks. On physical examination, she has a temperature of 37.6° C and an erythematous malar skin rash. Initial laboratory studies are positive for ANA at 1:1600 and anti-double-stranded DNA antibodies at 1:3200, along with false positive serologic test for syphilis. Serum creatinine is markedly elevated, and serum complement levels are decreased. In vitro tests of coagulation (prothrombin time and partial thromboplastin time) are prolonged. For which of the following conditions is she at greatest risk? A Acute pyelonephritis B Cerebral hemorrhage C Renal cell carcinoma D Malignant hypertension E Recurrent thrombosis

E The serologic features of systemic lupus erythematosus (SLE) include the more sensitive ANA test and the more specific anti-dsDNA test. The abnormal coagulation tests suggest the presence of anticardiolipin antibodies. These antibodies against phospholipid-protein complexes (antiphospholipid antibodies) also are called lupus anticoagulants because they interfere with in vitro clotting tests. In vivo, they are thrombogenic. Hence these patients can have recurrent thrombosis. Lupus anticoagulants also can occur in the absence of SLE. The other listed options are unlikely to be common or associated complications of SLE.

A 19-year-old man falls and strikes his leg. He feels intense pain. On physical examination there is swelling in his lower leg. MR imaging is performed and shown in the figure with T1 weighting. Which of the following is the most likely diagnosis? A Chondrosarcoma B Ewing sarcoma C Giant cell tumor D Metastatic seminoma E Osteosarcoma

E This osteosarcoma is a large destructive lytic and blastic mass arising in the metaphyseal region of the distal femur and extending into the surrounding soft tissue. This is the most common location. Trauma may call attention to this tumor, but is not a factor in pathogenesis. Chondrosarcomas arise over a wide age range, most often in the pelvis, shoulder, and ribs, and most patients are over 40 years of age. Ewing sarcomas arise in the diaphyseal region. Giant cell tumors also arise about the knee, but are large lytic, eccentric lesions with a thin rim of reactive, sclerotic bone. Such a large mass as that shown in the figure is unlikely to be a metastasis at any age; although testicular seminomas can occur in young men, they most often metastasize to regional lymph nodes.

A 33-year-old man has smoked two packs of cigarettes per day since he was a teenager. He has had painful thromboses of the superficial veins of the lower legs for 1 month and episodes during which his fingers become blue and cold. Over the next year, he develops chronic, poorly healing ulcerations of his feet. One toe becomes gangrenous and is amputated. Histologically, at the resection margin, there is an acute and chronic vasculitis involving medium-sized arteries, with segmental involvement. Which of the following is the most appropriate next step in treating this patient? A Antibiotic therapy B Corticosteroid therapy C Hemodialysis D Insulin therapy E Smoking cessation

E Thromboangiitis obliterans (Buerger disease), which affects small to medium-sized arteries of the extremities, is strongly associated with smoking. This disease may eventually involve adjacent peripheral veins and nerves. Syphilis can be treated with antibiotics, but it mainly produces an aortitis. Immunosuppressive therapy is not highly effective. Renal involvement does not occur. Although peripheral vascular disease with atherosclerosis is a typical finding in diabetes mellitus, vasculitis is not.

A 20-year-old man has noted a cluster of small lesions on his upper lip for the past 5 days. On physical examination, there are four lesions ranging from 0.2 to 0.5 cm that are raised and filled with clear fluid. Which of the following descriptive terms best applies to his lesions? A Bullae B Macules C Papules D Pustules E Vesicles

E Vesicles have a maximum size of 5 mm. They are often easily traumatized and unroofed to leave shallow ulcerations. In this case, herpetic vesicles are most likely present. Bullae are also fluid filled, but larger than 0.5 cm. Macules are not raised. They are flat areas less than 0.5 cm and distinguished by a difference in color; if the area is larger than 0.5 cm, then it is a patch. Papules are elevated and less than 0.5 cm; if larger, the term nodule is applied; either may be dome-shaped or flat-topped. Pustules are raised and filled with leukocytes to impart a yellow color.

A 12-year-old girl has had sudden onset of severe pain in her left knee that has awakened her from sleep on several occasions during the past 6 weeks. For each episode, her mother has given her acetylsalicylic acid (aspirin), and the pain has been relieved. On physical examination, there are no remarkable findings. A radiograph of the left knee shows a well-defined, 1-cm lucent area surrounded by a thin rim of bony sclerosis located in the proximal tibial cortex. The patient undergoes radioablation of the lesion, and the pain does not recur. What is the most likely diagnosis of this lesion? A Enchondroma B Fibrous dysplasia C Giant cell tumor D Osteoblastoma E Osteochondroma F Osteoid osteoma

F An osteoid osteoma is a benign tumor of the bone with a central nidus of woven bone and sclerotic rim. It most often occurs in children and young adults. Pain disproportionate to the size of the tumor is characteristic. If such a lesion is larger than 2 cm, it is classified as an osteoblastoma, and most of these arise in vertebral posterior elements. It can be treated effectively by curettage. The acute pain is mediated by release of prostaglandins, so aspirin is an effective analgesic. An enchondroma is a benign tumor of hyaline cartilage that arises in the medullary space in young adults. Fibrous dysplasia is a localized area of developmental arrest of bone formation. A giant cell tumor is a benign but locally aggressive lesion that arises in the epiphysis of the long bones of young adults and has a "soap bubble" radiographic appearance. An osteochondroma is a projection of the cartilaginous growth plate to form an exostosis.

A 10-year-old girl has a routine check of her health status. On physical examination, a flat, uniformly brown, 2-cm skin lesion is noted just above the right buttock. The lesion has been present since birth and has not changed in appearance. What is the most likely diagnosis of this lesion? A Acanthosis nigricans B Basal cell carcinoma C Dysplastic nevus D Lentigo E Malignant melanoma F Melanocytic nevus G Seborrheic keratosis

F The figure shows both junctional (along the dermal-epidermal junction) and intradermal nevus cells in a form of melanocytic nevus known as a congenital nevus. Larger nevi do have an increased risk of melanoma arising within them; however, the lack of additional lesions and the bland microscopic appearance shown here argue against dysplastic nevus syndrome. Acanthosis nigricans is an uncommon condition with hyperpigmented areas in skin folds; it may occur in association with endocrinopathies or with neoplasms. Basal cell carcinomas occur in sun-exposed skin of adults, and the cells have darker nuclei with scant cytoplasm. A lentigo is a common focal pigmented lesion that can appear at any age; it is a melanocyte hyperplasia. Melanoma occurs in adults and shows signs of rapid growth and change, with very atypical cells invading the dermis. Seborrheic keratoses are seen in older adults and are raised, pigmented lesions of thickened epidermis.

A 23-year-old man and other members of his racquetball club have noticed more itching of their feet in the past 2 months. On physical examination, the man has diffuse, erythematous, scaling skin lesions between the toes of both feet. There are no other remarkable findings. These findings are most likely the result of infection with which of the following organisms? A Group A β-hemolytic streptococcus B Herpes simplex virus C Human papillomavirus D Propionibacterium acnes E Sarcoptes scabiei F Staphylococcus aureus G Trichophyton rubrum

G Athlete's foot is a common disorder resulting from superficial dermatophyte infection by various fungal species, including Trichophyton, Epidermophyton, and Microsporum. Infections that involve the foot produce the condition known as tinea pedis. Streptococcal and staphylococcal organisms cause impetigo, which is more common on the face and hands. Herpetic infections first produce crops of clear vesicles, which may burst and form painful shallow ulcers. Human papillomavirus is best known as the cause of genital warts (condyloma acuminatum) and as a driving force behind cervical and anal squamous cell dysplasias. Propionibacterium acnes is a factor in the development of acne. The little eight-legged critters known as Sarcoptes scabiei crawl around in the stratum corneum, usually between the fingers, and cause itching, a process called scabies.

A 26-year-old man has noticed slowly enlarging lesions on his hands for the past 3 years. On physical examination, the lesions appear similar to those shown in the figure. There are no other remarkable findings. The lesions have not changed in color, do not itch or bleed, and are not associated with pain. What is the most likely diagnosis? A Condyloma acuminatum B Dermatofibroma C Intradermal nevus D Molluscum contagiosum E Seborrheic keratosis F Squamous cell carcinoma G Verruca vulgaris

G The warts in this patient are a common problem and result from infection with one of many types of human papillomavirus (HPV). They do not become malignant, and they tend to regress after several years. Condylomata acuminata, or genital warts, are caused by a type of HPV that is sexually transmitted; the lesions tend to be pink to white. A dermatofibroma forms a subcutaneous nodule, as does an intradermal nevus. Molluscum contagiosum is a self-limited condition with a nodular appearance. Seborrheic keratoses are pale brown to dark brown, slowly enlarging lesions with a rough surface that are most commonly found on the trunk and face of older individuals. A squamous cell carcinoma continues to grow irregularly, invade adjacent tissues, and ulcerate.


Set pelajaran terkait

lesson 7 & 8 strategic management

View Set

Color and the Electromagnetic Spectrum

View Set

Ch.14 Lesson 2: Adventure and Profit

View Set

Computer System Management Final Exam Review

View Set

Live Virtual Machine Lab 10.3: Module 10 Physical Network Security Concepts

View Set